Calculis and Diff Eqn

You might also like

Download as pdf or txt
Download as pdf or txt
You are on page 1of 157

B.Sc.

MATHEMATICS – I YEAR
DJM1A : CALCULUS AND DIFFERENTIAL EQUATIONS
SYLLABUS

UNIT -1

Curvature – radius of curvature - Cartesian and polar - centre of curvature - Involute


and evolute - Asymptotes in Cartesian and polar co-ordinates.

UNIT – 2

Evaluation of double and triple integrals-Jacobeans, change of variables.

UNIT -3

First order differential: equations of higher degree- solvable for p, x and y- Clairaut’s
form/ linear differential equations of second order- Particular integrals for functions of the
form, Xn, eax, eax(f(x)). Second order differential equations with variable coefficients.

UNIT – 4

Laplace transform – Inverse transform – Properties-Solving differential equations.


Simultaneous equations of first order using Laplace transform.

UNIT – 5

Partial differential equations of first order – formation – different kinds of solution –


four standard forms- Lagranges method.

Books :

1) Calculus Vol.1, Vol.2 & Vol.3, By T. K. Manickavachagompillai & others.

2) Calculus Vol.1, & Vol.2, By S. Arumugam and Isaac.

Manonmaniam Sundaranar University, Directorate of Distance & Continuing Education, Tirunelveli.


1
DJM1A : CALCULUS AND DIFFERENTIAL EQUATIONS

CONTENTS

UNIT -1
1.1 Curvature …………………………………… 1
1.2 Radius of curvature ………………………………….... 2
1.3 Cartesian and polar …………………………………… 2
1.4 Centre of curvature …………………………………… 9
1.5 Involute and evolute …………………………………… 10
1.6 Asymptotes in Cartesian and polar co-ordinates ………………… 24

UNIT – 2
1.1 Evaluation of double integrals ………………………………… 43
1.2 Triple integrals ………………………………… 51
1.3 Jacobians, ………………………………… 55
1.4 Change of variables. ………………………………… 59

UNIT - 3
1.1 First order differential: equations of higher degree
solvable for p, x and y ……………………………….. 65
1.2 Clairaut’s form/ ……………………………….. 69
1.3 Linear differential equations of second order ……….………. 72
1.4 Particular integrals for functions
of the form, Xn, eax, eax(f(x)). ………………………………. 77
1.5 Second order differential equations with variable coefficients……... 90

UNIT – 4
1.1 Laplace transform and property ……………………… 99
1.2 Inverse transform and property ……………………… 109
1.3 Simultaneous equations of first order using Laplace transform……. 124

UNIT – 5
1.1 Partial differential equations of first order …………………..….. 133
1.3 Four standard forms- ………………..…….. 140
1.4 Lagranges method. ………………..…….. 146
Manonmaniam Sundaranar University, Directorate of Distance & Continuing Education, Tirunelveli.
2
UNIT -1 : CURVATURE
Curvature – radius of curvature - Cartesian and polar - centre of curvature - Involute and
evolute - Asymptotes in Cartesian and polar co-ordinates.

CURVATURE

1. 1 Curvature and radius of curvature

The curvedness of a curve at a point  on it is measured by the rate of change of Ψ

with respect to s, where Ψ is the angle made by the tangent at  with the x-axis and s is the

arcual distance of  from a fixed point Q on the curve, that is by dΨ/ds.

This rate is called the curvature of the curve at .

Curvature of a circle

Consider a circle as in the figure whose centre is C and radius a. Let Ψ be the angle

made by the tangent at any point  with the x-axis. If the arcual distance of  from O is s,

then s = aΨ. This is the intrinsic eqn of the circle.

Differentiating this w.r.t ‘s’, we get

d
1 a .
ds
d 1
 
ds a

So, in the case of circle, the curvature is a constant which is the reciprocal of the
radius.

Manonmaniam Sundaranar University, Directorate of Distance & Continuing Education, Tirunelveli.


3
1.2 Radius of curvature

The reciprocal of Curvature of a curve at a point is called the radius of curvature of


ds
the curve at the point. So it is .
d

The radius of Curvature of a circle is its radius.

Notation

Radius of Curvature is denoted by .

Remark :1

d ds
In the case of a straight line the change of Ψ is zero and hence  0,   
ds d

Remark : 2

d
If the curve is such that, as ‘s’ increases, Ψ increases, then is +ve and, so  is +ve.
ds

ie) if the curve is concave,  is +ve otherwise is –ve In general,  is given as its absolute
value, namely  .

1.3. Cartesian formula for the radius of curvature

dy
We know that  tan 
dx

d2y d d ds
 2
 sec2 .  sec2  .
dx dx ds dx

Manonmaniam Sundaranar University, Directorate of Distance & Continuing Education, Tirunelveli.


4
ds sec3  dx
  as  cos 
d d 2y ds
dx 2

(1  tan 2  )
3
2


d2y
dx 2

3
  dy  2 
2

1    
  dx  

d2y
dx 2

3
  dy  2 
2

1    
  dx  
  
d2y
dx 2

Examples:

1. What is the radius of curvature of the curve x4 + y4 = 2 at the point (1,1)?

Soln:

Given the curve x4 + y4 = 2

Differentiating the above equation, we get

dy
4 x3  4 y 3  0.
dx

dy
4 x3  4 y 3 .
dx

Manonmaniam Sundaranar University, Directorate of Distance & Continuing Education, Tirunelveli.


5
dy x3
  3 .
dx y

Differentiating this once again, we get

 dy 
2 3 x 3  x2 y 
  .
d y dx
2 4
dx y

dy d2y
At the point (1,1),   1, and 2  6.
dx dx

(1  1) 2
3
2
   .
6 3

x
2. Show that the radius of curvature at any point of the catenary y  c cosh is equal to the
c
length of the portion of the normal intercepted between the curve and the axis of x.

Soln:

x
Given y  c cosh
c
Differentiating the above equation, we get

dy x
 sinh
dx c

3
  dy  2 
2 3
 x 2 x
Now,` 1      1  sinh 2   cosh 3
  dx    c c

d2y 1 x
Also 2
 cosh .
dx c c

Manonmaniam Sundaranar University, Directorate of Distance & Continuing Education, Tirunelveli.


6
x
cosh 3 2
Here   c  c cosh 2 x  y
1 x c c
cosh
c c

Again at any point (x,y)

1
  dy  2 
2
x y2
the normal  y 1      y cosh 
  dx   c c

 Radius of curvature = length of the normal.

3. If a curve is defined by the parametric equation x=f () and y=(), prove that the
1 x' y' ' y' x' '
curvature is 
 x'2  y'2 2
3

Soln:

where dashes denote differentiation with respect to .

dy dy dx y '
  
dx d d x'

d 2 y d  y '  d  y '  d
    
dx 2 dx  x'  d  x'  dx

y ' ' x' y ' x' ' 1



x '2 x'

y ' ' x' y ' x' '



x' 3

Manonmaniam Sundaranar University, Directorate of Distance & Continuing Education, Tirunelveli.


7
d 2y
1 dx 2 y ' ' x' y ' x' '
  

3 3
  dy  2   y '2 
2 2

1     x' 1  2 
3

  dx    x' 

x' y ' ' y ' x' '


 3 .
( x '2  y '2 ) 2

4. Prove that the radius of curvature at any point of the cycloid x =a ( + sin) and

y = a (1 - cos) is 4 a cos .
2
Soln:

From the given equations ,

x =a ( + sin)

differentiation with respect to .

dx
 a (1  cos  )
d
d 2x
 a sin 
d 2

y = a (1 - cos)

differentiation with respect to .

dy
 a sin 
d

d2y
 a cos  .
d 2

Manonmaniam Sundaranar University, Directorate of Distance & Continuing Education, Tirunelveli.


8
Substituting the values in the formula obtained in the previous example, we get

1 a(1  cos  ) a cos   a sin  ( a sin  )



 a (1  cos )
2 2
 a 2 sin 2  
3
2

a 2 (1  cos  )

a 3 2 (1  cos  ) 2
3

2 cos 2  / 2 1
 

a 4 cos 2  / 2  3
2

4 a cos 2


  4 a cos .
2

5. Find  at the point ‘t’ of the curve x = a (cos t + t sin t); y = a (sin t – t cos t)

Soln:

Given the curve

x = a (cos t + t sin t); y = a (sin t – t cos t)

dx
 a ( sin t  sin t  t cos t )  at cos t.
dt

dy
 a (cos t  cos t  t sin t )  at sin t.
dt

dy
  tant.
dx

Differentiating with respect to x,

Manonmaniam Sundaranar University, Directorate of Distance & Continuing Education, Tirunelveli.


9
d2y d dt 1 1
2
 (tan t )  sec 2 t 
dx dt dx at cos t at cos 3 t

3
  dy  2 
2

1  tan 2 t )
3

1    
2

  dx  
    1  at.
d2y at cos 3 t
dx 2

(The formula of Ex.3 can also be employed)

Exercise 1:

1. Find the radius of curvature for the curves


(a) y = ex at the point where it crosses the y – axis
(b) x  y  1at (1 / 4,1 / 4)
(c)y2 = x3 + 8 at the point (-2, 0).
(d) xy = 30 at the point (3,10)
(e) (x2 + y2)2 = a2 (y2 – x2) at the point (0, a)

Polar form.

Let r = f() be the given curve in polar coordinates.

 x = r cos  and y = r sin , may be regarded as the parametric equations of the


given curve the parameter being .

dx dr
  cos   r sin 
d d

dy dr
and  sin   r cos 
d d
Manonmaniam Sundaranar University, Directorate of Distance & Continuing Education, Tirunelveli.
10
d 2x d 2r dr
  cos   2 sin   r cos  and
d 2
d 2
d

d2y d 2r dr
 sin   2 cos   r sin 
d 2
d 2
d

Substituting these values in the formula for  in parametric from and simplifying we
get

3
(r 2  r1 ) 2
2
dr d 2r
 2 where r  and r  .
r  2r1  rr2 d d 2
2 `1 2

1.4 The coordinates of the centre of curvature

Let the centre of curvature of the curve y = f ( x) corresponding to the point P (x,y)
be X and Y.

X = ON

= OQ – NQ = OQ – MP

= X - PC sin Ψ = x -  sin Ψ.

Y = NC

= NM + MC

= QP + PC cos Ψ = y +  cos Ψ.

dy d2y
If y1 and y2 denote and we know that
dx dx 2
Manonmaniam Sundaranar University, Directorate of Distance & Continuing Education, Tirunelveli.
11
(1  y12 )
3
2

  and tan  y1

1 y1
cos  and sin 
1  y12 1  y12

(1  y12 ) 2 y1 (1  y12 )
3
y1
X  X   x
(1  y12 ) 2
1
y2 y2

(1  y12 ) 2 (1  y12 )
3
1
Y  y  y
(1  y12 ) 2
1
y2 y2

The locus of the centre of curvature for a curve is called the evolute of the curve.

Examples.

1. Find the co-ordinates of the centre of curvature of the curve xy = 2 at the point
(2,1).

Soln:

Given the curve xy = 2

2
Here y =
x
Differentiating with respect to ‘x’ we get

dy 2 d2y 4
  2 and 2  3 .
dx x dx x

dy d2y
 At (2,1) the values of and 2 arerespectively  1 / 2 and 1 / 2.
dx dx

Manonmaniam Sundaranar University, Directorate of Distance & Continuing Education, Tirunelveli.


12
1 1
(1  ) 
X 2 4 2  31.
1 4
2

1 1
Y  1 4  31.
1 2
2

1 1
 The centre of curvature is (3 ,3 ).
4 2

2. Show that in the parabola y2 = 4ax at the point t,  = - 2a (1+t2)3/2, X=2a + 3a t3,
Y = -2a t3. Deduce the equation of the evolutes.

Soln:

x = a t2, y = 2 at.

dx dy
  2at ,  2a
dt dt

dy 2a 1
  
dx 2at t

d 2 y d  dy  d  1  dx
 2      
dx dx  dx  dt  t  dt
1 1
 2
 2at  
t 2at 3

3
  dy  2 
2
d2y
  1      2   2a (1  t 2 ) 2
3

  dx   dx

Manonmaniam Sundaranar University, Directorate of Distance & Continuing Education, Tirunelveli.


13
2
 dy  dy  1 1
1   1  2 
X x  2   at 2  
dx dx t t
d y 1

dx 2 2at 3

 2a  3at 2

2
 dy  1
1   1 2
Y  y   2   2at 
dx t  2at
d y 1

dx 2 2at 2

Eliminating t from X and Y

3
 X  2a 
2

y   2a  
 3a 

Squaring both sides and simplifying, we get

27a Y 2  4 ( X  2a)3

The locus of (X,Y) is 27ay2 = 4 (x-2a)3

The curve is called a semi – cubical parabola.

x2 y2
3. Find the evolute of the ellipse   1 .Any point on the ellipse is (a cos , b sin )
a2 b2

Soln:
dx
x  a cos  ;   a sin 
d
dy
y  b sin  ;  b cos 
d

Manonmaniam Sundaranar University, Directorate of Distance & Continuing Education, Tirunelveli.


14
d2y d b b d
2
 ( cot  )  cos ec 2
dx dx a a dx

d 1

dx  a sin 

b
 2
cos ec 3
a

  dy  2  dy
1    
  dx   dx
X x
d2y
dx 2

 b2  b 
1  2 cot 2   cot  
 a cos   
a  a 
b
cos ec 3
a2

(a 2  b 2 ) cos 3 

a

2
 dy 
1  
Y  y  2 
dx
d y
dx 2

b2
1 cot 2 
 b sin   a2
b
2
cos ec 3
a

a 2  b2
 sin 3  .
b

Manonmaniam Sundaranar University, Directorate of Distance & Continuing Education, Tirunelveli.


15
1
 aX 
3

cos    2 2 
and
 a b 

1
  bY 
3

sin    2 2 
 a b 

To eliminate , squaring and adding, we get

2
2 3

 ax    by 
3

 2   2 2 
1
 a b   a b 
2

2
2 3

 ax   by 
3

i.e.,  2   2 2 
1
 a b   a b 
2

The locus of (X,Y) is the four cusped hypocycloid.

(ax) 3  (by) 3  (a 2  b 2 )
2 2 2
3

4. Show that the evolute of the cycloid

x  a(  sin  ); y  a(1  cos  )is another cycloid .

Soln:

Given
x  a(  sin  )

Differentiating with respect to θ


dx
 a(1  cos  )
d

Manonmaniam Sundaranar University, Directorate of Distance & Continuing Education, Tirunelveli.


16
y  a(1  cos  )

Differentiating with respect to θ

dy
 a sin  .
d

dy a sin  
   cot .
dx a(1  cos  ) 2

d2y d    1  d
  cot    cos ec 2 .
dx 2
dx  2 2 2 dx

1


4a sin 4
2

 
(1  cot 2 ) cot
X  x 2 2
1

4 a sin 4
2

 a(  sin  )  2a sin 

a(  sin  )


1  cot 2
Yy 2
1


4a sin 4
2

 a(1  cos  )  2a (1  cos  )

Manonmaniam Sundaranar University, Directorate of Distance & Continuing Education, Tirunelveli.


17
  a(1  cos )

 The locus of (X,Y) is

x  a (  sin  ); y  a(1  cos )

This is also a cycloid.

5. Find the centre of curvature of y = x2 at the origin.

Solution. We have y = x2.

 y1  2 x and y2  2

 At (0,0), y1  0 and y2  2

Let (x, y) be the centre of curvature at (0,0).

y1
 X x (1  y12 )  0.
y2

1  y12 1
Yy 
y2 2

1
 Centre of curvature is (0, ).
2

6. Find the evolute of the curve given by x  a cos 3  and y  sin 3  .

Solution. We have x  a cos 3  and y  sin 3  .

 y1   tan and y 2  (1/ 3a) sec4  cos ec 

Let (X, Y) be the centre of curvature.

Manonmaniam Sundaranar University, Directorate of Distance & Continuing Education, Tirunelveli.


18
y1 3a tan  (1  tan 2  )
X  x (1  y12 )  a cos 3  
y2 sec4  cos ec 

 a cos 3   _ 3a sin 2  cos  ......[1]

1  y12 3a tan  (1  tan 2  )


Yy  a sin 3  
y3 sec4  cos ec 

 a sin 3   3a cos 2  sin  ......[2]

Now, to find the equation of the evolute, we have to eliminate  from [1] and [2] we
have

X  Y  a(cos   sin  )3 .

X  Y  a(cos   sin  )3 .
2 2 2 2
( X  Y )  ( X  Y )  a (2)  2a
3 3 3 3

2 2 2
The locus of ( X , Y ) is ( x  y) 3  ( x  y) 3  2a 3 .

7.Find the evolute of the parabola y 2  4ax

Solution. We have y 2  4ax

 y1  2a / y and y2   4a 2 / y3

Let (X, Y) be the centre of curvature.

y1 y 2  4a 2
X x (1  y12 )  x 
y2 2a
 3x  2a (by 1) .....[2]

Manonmaniam Sundaranar University, Directorate of Distance & Continuing Education, Tirunelveli.


19
1  y12 y3
Y  y  2
y2 4a
3
2
2x
 (by 1) .....[3]
a

From[2] and [3] eliminating x , we have

4 x 3 4( X  2a)3
Y 
2

a 27a

27aY 2  4 ( X  2a)3

 The locus of ( X , Y ) is 27 ay 2  4 ( x  2a)3

Theorem 8 The normal to a given curve is tangent to its evolute.

Proof. We know that the coordinates of the centre of curvature of the given curve are given
by

y1
X x (1  y12
y2

1  y12
Y y
y2

These two equations can be taken as the parametric equation of the evolute with x as
parameter.

dX y  
 y 2  y1 y3 

 1   1  2 y1 y2  (1  y12 )  2 
dx  y2  
 y 22 

Manonmaniam Sundaranar University, Directorate of Distance & Continuing Education, Tirunelveli.


20
y1 y3
1  2 y12  (1  y12 )(1  2
)
y2
3
y1 y3 y1 y3
  3 y1  
2
2 2
)
y2 y2

y1`
 (3 y3  y3  y1 y3 ).
2 2
2
y2

dY 
 2 y1 y22  (1  y1 ) y3 
2
 2
 y1   3 y3  y3  y1 y3
2
Now 4
dx 
 y2 

1
 (3 y1 y2  y3  y1 y3 )
2 2
2
y2
dY 1
  ….(1)
dX y1

dY
But is the slope of the tangent to the evolute and y1 is the slope of the tangent to
dX
the given curve at the corresponding point and their product is -1 (by 1).

 Tangent to the evolute is normal to the given curve.

Exercises

1. Find the coordinates of the center of curvature at the indicated points.

1 1
[a] y = x2 at ( , )
3 4
[b] xy = c2 at (c,c).
[c] x = a (cos t + t sin t), y=a (sin t – t cos t) at ‘t’.
[d] y = x log x at the point where y’ = 0.

Manonmaniam Sundaranar University, Directorate of Distance & Continuing Education, Tirunelveli.


21
Exercise 2:

1. Find the coordinates of the centres of curvature at given points on the curves :

(1) y  x 2 ; ( 1 , 1 )
2 4
(2) xy  c ; (c, c).
2

 
(3) y  log sec x;  , log 2 
3 

2. Prove that the circle of curvature at the point (t2, 2t) of the curve y2=4x cuts the curve
again at a point whose ordinate is – 6t. Calculate the coordinates of the centre of
curvature.

1.5 Evolute and involute.

We have already defined evolute of a curve as the locus of the centre of curvature and

deduced the equations of the evolute of the parabola and ellipse.

If the evolute itself be regarded as the original curve, a curve of which it is the evolute

is called an involute.

It may be noted that there is but one evolute but an infinite number of involutes.

Radius of curvature when the curve is given in polar co-ordinates

Let us assume that the equation of the curve in polar coordinates be r = f ().

In the figure,

    .

Manonmaniam Sundaranar University, Directorate of Distance & Continuing Education, Tirunelveli.


22
d d
  1 .
d d

We have proved that

d r
tan   r  .
dt  dr 
 
 d 

Differentiating w.r.t , we get

2
 dr  d 2r
  r 2
d  d  d
Sec 2 
d  dr 
2

 
 d 

2
 dr  d 2r
  r 2
d 1  d  d
 
d sec 
2
 dr 
2

 
 d 

2
 dr  d 2r
  r 2

1  d  d
2 2
1
r  dr 
 
 d 
2
 dr 
 
 d 

2
 dr  d 2r
  r 2
d  d
 2
 dr 
  r
2

 d 

d d
1 
d d

Manonmaniam Sundaranar University, Directorate of Distance & Continuing Education, Tirunelveli.


23
2
 dr  d 2r
  r 2
d  d
1   2
 dr 
  r
2

 d 

2
 dr  d 2r
r  2
2
 r 2
  d  d
2
 dr 
  r
2

 d  

We have proved in the previous chapter that

ds   dr  
2 2
 
  r 2    
d   d  

ds ds d
  .
d d d

2
 dr 
  r
1 2
  dr  
2
 d 
2
 
 r 2    
  d    dr   d r 
2 2
  r  2
2
   r 2 
 d   d 

3
 2  dr  2 
2

r    
  d  
 2
 dr  d 2r
r  2
2
 r 2
 d  d

Examples .

1. Find the radius of curvature of the cardioid r = a (1-cos).

Manonmaniam Sundaranar University, Directorate of Distance & Continuing Education, Tirunelveli.


24
Soln:
Given r = a (1-cos).

Differentiating w.r.t , we get

dr d 2r
 a sin  , and 2  a cos  .
d d

3

 2  dr  
 
2 2
 3
 r      a (1  cos  )  (a sin  )
2 2 2 2 2


  d  

 8a 3 sin 3 .
2
2
 dr  d 2r
r 2  2   r 2  a (1  cos  )  2a sin   a cos  (1  cos  )
2 2 2 2 2

 d  d


 6 a 2 sin 2
2


8a 3 Sin 3
  2  4 a sin 
 3 2
6a 2 sin 2
2

2
 2 ar
3 .

2. Show that the radius of curvature of the curve

a n r  n 1
r n  a n Cos n is
n 1
Soln:

Taking logarithms on both sides and differentiating, we get

Manonmaniam Sundaranar University, Directorate of Distance & Continuing Education, Tirunelveli.


25
n dr n sin n

r d cos n

dr
   r tan n 
d

Differentiating once again w.r.t. , we get

d 2r dr
`   tan n   nr sec 2 n
d 3
d

 r tan 2 n  nr sec2 n

(r 2  r 2 tan2 n

r 2  2r 2 tan 2 n  r 2 tan 2 n  nr 2 sec2 n

r 3 sec3 n r
 
(n  1) r sec n (n  1) cos n 
2 2

r. a n a n r  n 1
 
(n  1) r n n 1

Particular cases.

i) Putting n = 2, we get Bernouilli’s lemniscate;


a2

3 r
ii) when n = -2 we have a rectangular hyperbola.

r3

a2

Manonmaniam Sundaranar University, Directorate of Distance & Continuing Education, Tirunelveli.


26
2
iii) when n = ½, we get cardioids;   ar
3

2r 3 / 2
iv) when n = -1/2, we get a parabola,  
a

a
v) when n =1. we get a circles ;  
2 .

1.6 Asymptotes in Cartesian and polar co-ordinates.

Asymptotes of polar curves.

1 1
Theorem: For the polar curves of the form  f ( ) the asymptotes are r sin(  i ) 
r f ' (i )

where i ’s are the roots of the equation f() = 0.

1
Proof: Let P (r,) be any point on the curve  f ( ) ………………… (1)
r
As P tends to infinity along the curve r  . Hence from (1) we note that
when r  , f()  0.

Let the roots of the equation f() = 0


be i where i = 1,2,3 ……….

Hence i ’s are the only directions along which the branches of the curve tend to infinity. Let
us consider the branch corresponding to the value  = 1.

For this branch   1, r  .

We know that the polar equation of any line is p = r cos( - )


where p is the length of the perpendicular from the pole on the line and  is the angle which
this perpendicular makes with the initial line and (r,) is any point on the line.
Manonmaniam Sundaranar University, Directorate of Distance & Continuing Education, Tirunelveli.
27
Consider an asymptote to the branch corresponding to  = 1.
Let it be p = r cos ( - ) ………………. (2)

For this branch corresponding to  = 1. the equation of the asymptote will be determined if
we find p and .

Draw ON perpendicular to the line (2).


Then ON = p and XON = .
Draw PM perpendicular to the line (2) and PL perpendicular to ON.
 PM = LN = ON - OL = p-OP cos( - )
= p - r cos( - ).
PM p
   cos(   ) ……………………… (3).
r r

Since (2) is an asymptote of (1), by definition, PM  0 as P  .

p
(i.e) As r  , f()  0 as   1 so that 0
r
PM p 
 From (3) lim  lim   cos(   )
 1 r   1  r 
(i.e) 0  0  lim cos(   )
 1


 0 = cos(1 - ). Hence 1    .
2


  1  . …………………………. (4)
2

1 cos(   )
Also using  f ( ) in (3) we get PM  p 
r f ( )

Manonmaniam Sundaranar University, Directorate of Distance & Continuing Education, Tirunelveli.


28
 cos(   ) 
Taking limit as   1 we have 0  lim  p 
 1
 f ( ) 

cos(   )
 p  lim
 1 f ( )

   
 cos  1   
cos(   )
 lim  
2
 p  lim  (Using (4))
 1 f ( )  1  f ( ) 
 

  sin(  1 ) 
 lim 
 1
 f ( ) 

 cos(  1 ) 
  lim  
 1
 f ' ( ) 

(Using L-Hospital’s rule)

 cos 0 
 
 f ' (1 ) 

1
p .
f ' (1 )

Substituting the values of p and  in (2) we get

1  
  r cos   1    r sin(  1 )
f ' (1 )  2

1
 r sin(  1 )  .
f ' (1 )

This is the required asymptote corresponding to  = 1.


Manonmaniam Sundaranar University, Directorate of Distance & Continuing Education, Tirunelveli.
29
Similarly the other asymptotes corresponding to the other roots of
f() = 0 can be got.

Working rule to find the asymptotes of the polar curves.

1
1. Write the polar equation in the form  f ( ).
r
2. Find the roots of f() = 0. Let the roots be 1,2,3…….

3. Find f() and calculate f() at  = 1,2,3…….

1 1
4. Then write the equations of the asymptotesas r sin(  1 )  , r sin(   2 )  ,
f ' (1 ) f ' ( 2 )
1
r sin(  3 )  .........
f ' (3 )

Solved problems.

Problem 1. Find the equations of the asymptotes of the following curves

(i) r  = a (ii) r log  = a.

1
Solution. (i): The equation of the given curve in the form  f ( )
r

1  
we have  so that f ( )  .
r a a


Now, f ( )  0   0. Hence   0.
a

1 1
Also, f ' ( )  . Hence f ' (0)  .
a a

Manonmaniam Sundaranar University, Directorate of Distance & Continuing Education, Tirunelveli.


30
1 1
 The equation of the asymptote is r sin(  0)   .
f ' (0) a
(i.e) r sin  = a.

1
(ii) The equation of the given curve in the form  f ( )
r
1 log  log 
we have  so that f ( )  .
r a a

log 
Now, f() = 0   0. Hence  = 1.
a

1 1
Also, f ' ( )  . Hence f' (1)  .
a a

1
 The equation of the asymptote is r sin(  1)   a.
f ' (1)
(i.e) r sin(-1) = a .

Problem 2. Find the equation of the asymptotes of the curve


r(2 - 2) = 2aθ

Soln: The equation of the given curve in the form


1
 f( )
r

1  2  2  2  2
we have  so that f ( )  .
r 2a 2a

 2  2
Now, f ( )  0  0
2a

 2 = 2.

Manonmaniam Sundaranar University, Directorate of Distance & Continuing Education, Tirunelveli.


31
  = .

1  2 2  ( 2   2 )   2   2
Also, f ' ( )    2a 2
2a  2 

 2  2 1 1
 f ' ( )   . Also f ' ( )  .
2a 2
a a

 The equation of the asymptote corresponding to  =  is

1
r sin(   )   a.
f ' ( )

(i.e) -r sin  = a. Hence r sin  + a = 0 ………..(1) Similarly the asymptote


corresponding to  = - is

1 1
r sin(   )   .
f ' ( ) a

(i.e) r sin  + a = 0 which is same as (1).

Hence there is only one asymptote for the given curve.

a
Problem 3. Find the equation of the asymptotes of the curve r 
1 cos 
1
Solution. The equation of the given curve in the form  f ( )
r

1 1  cos  1  cos 
we have  so that f ( )  .
r a a

1  cos 
Now, f ( )  0  0
a

Manonmaniam Sundaranar University, Directorate of Distance & Continuing Education, Tirunelveli.


32
 1 - cos  = 0
 cos  = 1
  = 2n0 where n ϵ Z
  = 2n where n ϵ Z.

sin  sin 2n 
Now, f ' ( )  . Hence f' (2n )   0 for all n Z .
a a
1
   for all n  Z
f ' (2n )

Hence the curve has no asymptotes.

Problem 4. Find the equation of the asymptotes of the curve r cos  = a sin .
1
Solution. Writing the equation of the given curve in the form  f ( ).
r

1 cos  cos 
we have  so that f ( )  .
r a sin  a sin 

Now, f() = 0  cos  = 0


   (2n  1) where n Z .
2

cot  cos ec 2 1
Since f ( )  we have f ' ( )   
a a a sin 2 

  1
Now, f ' (2n  1)   for all n  Z .
 2 2 
 a sin (2n  1) 
 2

1


a sin 2 (n  )
2
Manonmaniam Sundaranar University, Directorate of Distance & Continuing Education, Tirunelveli.
33
1 1 1
   .

a (1) n sin  / 2 
2
a(1) 2n
a

  1
 The asymptotes are given by r sin   (2n  1)   ,n Z
 2  
f ' (2n  1) 
 2

  
(i.e)  r sin (2n  1)     a
 2 

  
(i.e) r sin (2n  1)     a for all n  Z .
 2 

 
(i.e) r sin      a
2 

(i.e) r cos  = a

a
(i.e) r cos   .
(1) n

For different values of n ϵ Z we get only two asymptotes r cos  = -a and r cos  = a.
Hence the asymptotes are r cos  = a.

Exercises.

Find the asymptotes of the following curves.

a
(i) r  (ii) r(1 - e) = a
 1

Tracing of curves f(x,y) = 0 (Cartesian Coordinates)

Manonmaniam Sundaranar University, Directorate of Distance & Continuing Education, Tirunelveli.


34
Suppose a curve is represented in terms of Cartesian coordinates by the equation
f(x,y) = 0. The following points provide useful informations regarding the shape and nature
of the curve.

I. Symmetry of the curve.

(a) Symmetry about the x - axis:

A curve f(x,y) = 0 is symmetric about the x-axis if f(x,-y) = f(x,y).

Example. y2 = 4ax; x2 + y2 = a2; y4 + y2 + x3 = 0 are curves which are symmetric


about the x - axis.

But x2 + y2 = ay is not symmetric about the x-axis.

(b) Symmetry about the y - axis.

A curve f(x,y) = 0 is symmetric about the y - axis if f(-x,y) = f(x,y)

Example. x2 = 4ay; x2 + y2 = a2; y = x4 + x2 + a are symmetric about y - axis.


But x2 + y2 = ax is not symmetric about y - axis.

Note. x2 + y2 = a2 is symmetric about x - axis and y - axis. In this case the equation
involves even and only even powers of x as well as y.

(c) Symmetry about the line y = x

If f(x,y) = f(y,x) then the curve is symmetric about the line y = x.

Example. x2 + y2 = a2; x3 + y3 = 3axy; xy = c2 are symmetric about the line y = x.

Manonmaniam Sundaranar University, Directorate of Distance & Continuing Education, Tirunelveli.


35
(d) Symmetric about the origin. (Symmetric in opposite quadrants)

If f(-x,-y) = f(x,y) then the curve is symmetric about the origin (symmetric in opposite
quadrants)

Examples. x2 + y2 = a2; xy = c2 are symmetric about the origin.

x3 + y3 = 3axy; y2 = x3 are not symmetric about the origin.

Note. From the above examples the equation of the circle has all symmetric properties we
have discussed so far.

Points of intersection with the coordinate axes.

To obtain the points where the curve f(x,y) = 0 intersects the x - axis put y = 0 in the
equation and solve for x. Similarly, to find the points where the curve intersects the y - axis
put x = 0 in the equation and solve for y.

Examples. The curve x2 + y2 = a2 crosses the x - axis at (a,0) and (-a,0) and crosses
the y - axis at (0,a) and (0,-a).

The curve y2 = 4ax passes through the origin.

IV. Tangents to the curve.

(a) Tangents at the origin.

If the origin is found to be a point on the curve then the tangents at the origin are
obtained by equating to zero the lowest degree terms occuring in the equation.

Example. y2 = 4ax passes through the origin and the lowest degree term occuring in
it is 4ax which when equated to zero becomes 4ax = 0

Manonmaniam Sundaranar University, Directorate of Distance & Continuing Education, Tirunelveli.


36
(i.e) x = 0. Hence y - axis is the tangent to the parabola at the origin.

Also x3 + y3 = 3axy passes through the origin at which x = 0 and y = 0 are the
tangents.

For the curve a2y2 = a2x2 - x4, y = x are the tangents at the origin.

(b) Tangents at any other point (h,k) other than the origin.

dy
Find at (h,k) and it gives the slope of the tangent to the curve at this point. This
dx
will be useful to deside the nature of the tangent - whether parallel to the x - axis or y - axis or
inclined tangent.

V. Asymptotes.

The concept of asymptotes described in the previous chapter will be helpful to know
about the asymptotes in tracing any curve.

(a) Asymptotes parallel to the x - axis.

These are obtained by equating to zero the coefficient of the highest power of x.

Example. (y+a)x2 + x - 1 = 0 has an asymptote y = -a parallel to the x - axis.

(b) Asymptotes parallel to the y - axis.

These are obtained by equating to zero the coefficient of the highest power of y.

Example. y2(4 - x2) = x3 - 1 has asymptotes 4 - x2 = 0 (i.e) x = 2 and 𝜆 = -2 are two


asymptotes parallel to the y - axis.

Manonmaniam Sundaranar University, Directorate of Distance & Continuing Education, Tirunelveli.


37
(c) Inclined asymptotes.

Taking y = mx + c as an asymptote we can find m and c by substituting y = mx + c in


the equation and equating to zero the various powers of x starting from the highest power.

Polar Coordinates : (Tracing a curve f(r,θ) = 0):

To trace curve given in terms of polar coordinates by the equation f(r,θ) = 0.

I. Symmetry of the curve.

(a) Symmetry about the initial line:

The curve f(r,θ) = 0 is symmetric about the initial line θ = 0 if f(r, -θ) = f(r, θ).

Example. r = a(1+cosθ); r = a(1-cosθ); r = a cos2θ Symmetric about the initial line.

However r = a(1-sinθ) is not Symmetric about the initial line.

Symmetry about the pole


.
The curve is symmetric about the pole if f(-r,θ) = f(r, θ).

Example: r2 = a2 cos2θ ; r2 = a2 sin2θ are Symmetric about the pole.


.
𝝅
(c) Symmetry about θ = 𝟐

𝜋
The curve f(r,θ) = 0 is symmetric about the line θ = 2 (y-axis) if f(r, 𝜋, -θ) = f(r, θ). :

𝜋
Example : r = a(1+sinθ) ; r = a sin3θ are symmetric about θ = 2

Manonmaniam Sundaranar University, Directorate of Distance & Continuing Education, Tirunelveli.


38
Tangents at the pole.

We put r = θ in the equation of the curve and solve the resulting equation for θ.If there
exists a real solution α for θ,then the curve passes through the pole and the line θ=α is
a tangent to the curve at the pole.

Region in which the curve lies.

1) If the maximum value of r is a, then the curve lies within the circle r = a.

2) If there exist values of θ for which r2<0 so that r becomes imaginary then the curve
does not exist for those values of θ.

𝜋
Example: r2=a2 sin2θ does not exist if <θ<𝜋
2

Value of 𝜙.
The angle 𝜙 which a tangent at(r, θ) makes with the initial line is found from the
𝑑𝜃
formula tan𝜙= r 𝑑𝑟

Asymptotes:
If there is no finite value α for θ such that r͚  , then the curve f(r, θ) = 0 has no
asymptotes

Points on the curve:


Giving different values for θ we can get different points on the curve which will be of
use in tracing the curve and ascertain whether r increases or decreases in the region.

Tracing a curve x = f(t), y = g(t) (parametric equations)

(i) Suppose x = f(t), y = g(t) are parametric eqations of a curve where t is the parameter.

If it is possible to eliminate the parameter between the two equations and get the
Manonmaniam Sundaranar University, Directorate of Distance & Continuing Education, Tirunelveli.
39
Cartesian form of the curve we proceed as in the case of Cartesian coordinates.

(ii) If the parameter t can not be easily eliminated

𝑑𝑦 𝑑𝑦 𝑑𝑡
(a) Find 𝑑𝑥 = 𝑑𝑡 × 𝑑𝑥
𝑑𝑦
(b) Give different values to the parameter t and find x, y, 𝑑𝑥 . This gives different points

on the curve end slopes of the tangents at these points.


(c) We plot the points and trace the curve.

Solved Problems:

( x) 3  ( y) 3  (a ) 3 (four cusped cycloid or asteroid)


2 2 2
Problem 1 : Trace the curve
Solution:

Given the curve ( x)  ( y)  (a )


2 2 2
3 3 3
……………………….(1)

Clearly the curve is symmetrical about both the axes. Hence it is enough to discuss

The nature of the curve in the first guadrant only.

To find the points of intersection of the curve with x-axis, we put y=0 in eqn (1)

We get ( x) 3  (a 2 ) 3
2 2

Therefore x2 = a2 and hence x=± a.


Hence the curve meets the x-axis at (0,a) and (-a,0).
Similarly, the curve meets the y-axis at (a,0) and (0,-a).
2
2 3

 y  x
3

  1  
From eqn (1)  a   a  ,we see that if │x│> a,then
2
 y
3

 
a < 0 and hence y is imaginary.
Hence the curve does not lie beyond x = ± a
Similarly, the curve does not lie beyond y = ± a

Manonmaniam Sundaranar University, Directorate of Distance & Continuing Education, Tirunelveli.


40
 dy  y
Also,    3
 dx  x

3 3

 dy 
 0
 dx 

At (a, 0) and hence x-axis is a tangent to the two branches of the curve
at (a, 0) lying in the first and fourth quadrants.
Hence the curve has a cusp of first kind at (a, 0).
Similarly, the curve has cusps of first kind at (0, a), (-a, 0), (0,-a).
Hence the curve is known as four cusp hypocycloid.
Also, the curve is concave in [0,a].
Hence the form of the curve is as shown in the fig.

Note: The parametric equation of this curve can be taken as x= acos3θ; y = asin3θ .

Problem 2 : Trace the curve y2 (2a - x) = x3 (cissoid)

Solution. y2 (2a - x) = x3 ……………………………… [1]

Since [1] contains even power of y the curve is symmetrical about the x-axis.

Obviously it passes through the origin.

The tangents at the origin are given by y2 = 0 and they are real and coincident. Hence
the origin is a cusp.
The curve meets the x-axis and y-axis only at the origin.
Equating the coefficient of the highest degree term in y to zero we get x - 2a = 0. The
asymptotes parallel to the y - axis is x - 2a = 0 and this is the only asymptote to the curve.

Manonmaniam Sundaranar University, Directorate of Distance & Continuing Education, Tirunelveli.


41
x
Writing the given equation as y  x . (Considering the positive root) we see
2a  x
that y is imaginary when x < 0 or when x > 2a.

Hence the curve does not lie to the left of the y - axis and to the right of the line
x = 2a.

As x increases from 0 to 2a y increases from 0 to .

Hence the form of the curve is as shown in the figure and the curve is called cissoid.

Problem 3: Trace the curve r = a(1+cos ) where a > 0 (cardioid).

Solution. We note the following from the equation of the given curve. The curve is
symmetric about the initial line.

When  =  we have r = 0. Hence the curve passes through the pole and further  = 
is the tangent at the pole.

Let  be the angle made by the tangent at (r, ) with the initial line.

d a(1  cos  )     
Now, tan  r    cot    tan   .
dr  a sin  2  2 2

Manonmaniam Sundaranar University, Directorate of Distance & Continuing Education, Tirunelveli.


42
 0 
   Hence when  = 0,   and r = 2a.
2 2 2

Thus the tangent at (2a, 0) is perpendicular to the initial line.

Since the maximum value of r is 2a, no portion of the curve lies to the right of the
tangent at (2a, 0) and hence the curve lies within the circle r = 2a.

The following table gives a set of points lying on the curve.

 0 /4 /2  - /2 - /4

1 1
r 2a a (1+ ) a 0 a a (1+ )
2 2

When  increases from 0 to 2, r is positive and it decreases from 2a to 0. The form of
the curve is as shown in the figure and it is a cardioid.

Problem 4 : Trace the curve r2 = a2 cos 2  (Lemniscate of Bernoulli)

Solution. The curve is symmetric about the pole and the initial line.

Manonmaniam Sundaranar University, Directorate of Distance & Continuing Education, Tirunelveli.


43
Negative values of cos 2  give imaginary values of r. Hence the curve lies in the two
7  3 5
quadrants bounded by   ,  ; and   ,  .
4 4 4 4

 3 5 7
The lines   ,  ;  ,  are tangents to the curve at the people.
4 4 4 4

Some points on the curve are given below.

 0 /4 3/4 5/4 7/4


r A 0 0 0 0

7
When  increases from   to 2 (=0), r incerases from 0 to a and when 
4

increases from 0 to   r decreases from a to 0.
4

The form of the curve is as shown in the figure below.

Problem 5 : Trace the curve r = a(1-cosθ)

Soln: The curve is symmetric about the initial line.


Manonmaniam Sundaranar University, Directorate of Distance & Continuing Education, Tirunelveli.
44
It passes through the pole.

tan𝜙 = tan(θ/2). Hence 𝜙 = θ/2.


At (2a, 𝜋), the tangent is perpendicular to the initial line.

Problem 6 : Trace the curve r = sin 3 .


Solution. The curve is symmetric about the line  
2

r = 0  sin 3  = 0  3  = 0 or multiple of 

 2 3 4 5
  = 0, , , , , .
3 3 3 3 3

For  = 0 the curve passes through the pole.

 4 2 5
Further  = 0,  =  ;   ,  ; and   ,  are the tangents to the curve at the
3 3 3 3
pole

Since |sin 3θ| ≤1, r ≤ a. Hence the curve lies entirely within the circle r = a.

We get different points on the curve as shown in the table.


Manonmaniam Sundaranar University, Directorate of Distance & Continuing Education, Tirunelveli.
45
θ 0 π/6 π/3 π/2 2π/3 5π/6 π
r 0 a 0 -a 0 A 0

When 0 ≤ θ ≤ π/ 3 the curve has one loop around θ = π/6.

When 4π/3 ≤ θ ≤ 5π/ 3 the curve has another loop around θ = 3π/2.

When 2π/3 ≤ θ ≤ π the curve has yet another loop around θ = 5π/6.

The form of the courve as shown in the figure and it is known as three leaved rose.

Manonmaniam Sundaranar University, Directorate of Distance & Continuing Education, Tirunelveli.


46
UNIT – 2 : EVALUATION OF DOUBLE INTEGRALS
Evaluation of double and triple integrals-Jacobeans, change of variables.

EVALUATION OF DOUBLE INTEGRALS

1.1 Double Integrals:

Let f(x,y) be a continuous function defind on a closed rectangle R = {(x,y)/ a  x  b


and c  y  d}.
d
For any fixed x[a,b] consider the integral  f ( x, y)dy.
c

The value of this integral depends on x and we get a new function of x. This can be
b
d 
integrated with respect to x and we get c c f ( x, y) dy dx.
This is called an iterated integral.
d
b 
Similarly we can define another integral c a f ( x, y)dxdy.
For continuous functions f(x,y) we have


b d
 
d b


R
f ( x, y ) dx dy    f ( x, y ) dy dx    f ( x, y ) dx  dy
c c  c a 

We omit the proof of this result.

If f(x,y) is continuous on a bounded region S and if S is given by S = { (x,y)/ a  x  b


and 1(x)  y  2(x)} where 1 and 2 are two continuous functions definded on [a, b] then

𝑏 𝜑 2 (𝑥)
𝑆
𝑓 𝑥, 𝑦 𝑑𝑥 𝑑𝑦 = 𝑎 𝜑 1 (𝑥)
𝑓 𝑥, 𝑦 𝑑𝑦 𝑑𝑥

The inerated integral in the right hand side is also written in the form

Manonmaniam Sundaranar University, Directorate of Distance & Continuing Education, Tirunelveli.


47
𝑏 𝜑 2 (𝑥)
𝑎
𝑑𝑥 𝜑 1 (𝑥)
𝑓 𝑥, 𝑦 𝑑𝑦.

Similarly if S = { (x,y) / c  y  d and 1(y)  x  2(y)}


then
𝑑 𝜑 2 (𝑦)
𝑆
𝑓 𝑥, 𝑦 𝑑𝑥 𝑑𝑦 = 𝑐 𝜑 1 (𝑦)
𝑓 𝑥, 𝑦 𝑑𝑥 𝑑𝑦

If S cannot be written in neither of the above two forms we devide S into finite
number of subregions such that each of the subregion can be represented in one of the above
forms and we get the double integral over S by adding the integrals over these subregions.

Hence to evaluate  f ( x, y) dx dy we first convert it to an iterated integral of the two


D

forms given above.

Solved Problems.
1 2
Problem 1. Evaluate I    xy 2 dy dx.
0 0

Solution.

2 1
8  x2 
1 2
1 3 
1 1
1 4
Now  xy dy dx    xy  dx   8 x dx    
2

0 0
0 0
3 30 3  2 0 3

1 2
4
ThereforeI    xy 2 dy dx  .
0 0
3

4 a 2 ax
Problem :2 Evaluate I    xy dy dx.
0 x2 a
4

Manonmaniam Sundaranar University, Directorate of Distance & Continuing Education, Tirunelveli.


48
2 ax
 xy 2 
4a
Solution. I  
2  x 2 / 4 a
dx
0

4a
1
4a
 x4  1  4ax 3 x6  64a 4
=      
16a 2  2  3 96a 2  0
x 4 ax dx
20  3

 
a b
Problem : 3 Evaluate I    x 2  y 2 dx dy
0 0

b
a
 x3 2
a
 b3 
Solution. I     xy  dy     by 2  dy
0 0 0 
3 3

a
 b3 y by 3 
 
 3 3  0

= 1/3 ab (a2 + b2).

a  
b 1 x 2 / a 2 
Problem 4. Evaluate I   dx x
3
y dy
0 0

 
b 1 x 2 / a 2 
1 
a
Solution. I    x3 y 3  dx
0 0
2

a
1 a 2 3  x2  1 2  1 4 x6 
  b x 1  2  dx  b  x  2 
 
2 0  a  2 4 6a  0

a 4b 2
 .
24

1/ 2 1
x
Problem 5 : Evaluate I    1  x y  dy dx.
0 0
2 2

Manonmaniam Sundaranar University, Directorate of Distance & Continuing Education, Tirunelveli.


49
1
 sin  1( xy ) 
1/ 2
Solution. I   x  dx
0  0
x

 
1/ 2 1/ 2
  sin 1 xdx  x sin 1 x  1  x 2 0 (integration by parts)
0

π 3
   1.
12 2

π a cosθ
Problem 6 : Evaluate I    r sin θ dr dθ
0 0

π a cos θ
1 
Solution. I   sin θ  r 2  dθ
0  2 0

π π
1 1
  a 2 cos 2 θ sin θ dθ   a 2  cos 2 θ d (cos θ )
20 2 0

1 π
  a 2 cos 3 θ 0
6
 

= 1/3 a2

π/2 
r
Problem 7. Evaluate   r
0 0
2
 a2 
2
dr dθ

π/2 
r
Solution Let I    r
0 0
2
 a2 
2
dr dθ

 1 2  
π/2 
 2 d (r )  π/2
1  1 
   2 2 2
dθ    2
2  r  a 2  0

0  0 (r  a )  0
 

π/2 π/2
1 dθ  θ 

2 
0

a 2  2a 2  0

Manonmaniam Sundaranar University, Directorate of Distance & Continuing Education, Tirunelveli.


50
π
 .
4a 2

Change of order of Integration:

Problem 1. Evaluate I   xy dy dx where D is the region bounded by the curve x = y2,


D

x = 2-y, y=0 and y = 1.

Solution. The given region bounded by the curves is given in the figure.

In this region x various from 0 to 2. When 0  x  1, for fixed x

y varies from 0 to x. when 1 ≤ x ≤ 2,y various from 0 to 2-x.

The region D can be subdivided into two regions D1 and D2 as shown in the figure.

 xy dx dy   xy dy dx   xy dy dx
D D1 D2

1 x 2 2 x
   xy dy dx    xy dy dx
0 0 1 0

x 2 x
1  1 
1 2
   xy 2  dx    xy 2  dx.
0  0 1  0
2 2
1 2
1 1
  x 2dx   x(2  x)2dx
20 21
1 2
1  1  1 4 
  x 3   2 x 2  x 4  x 3 
 6 0 2  4 3 1

1 1 1 4 1 4
=  [(8   16   8)  (2   )]
6 2 4 3 4 3
=9/24 (verify).

Manonmaniam Sundaranar University, Directorate of Distance & Continuing Education, Tirunelveli.


51
 x y 2 dxdy where D is the circular disc x2+y2  1.
2
Problem 2. Evaluate
D

Solution. In D, x varies from -1 to 1. For a fixed x. y varies from - 1  x 2  to 1  x 


2

1 1 x 
2

  x y dx dy  
2 2
x
2
y 2 dy dx
D 
1  1 x 2 
1 1 x 
2

 4 x
2
y 2 dy dx
0 0

1 x 
2

1 
1
 4  x 2 y 3  dx
0  0
3
1
 4 x 2 (1  x 2 )3 / 2dx
0

π/2
4
 
3 0
sin 2 θ cos 4 θ dθ (Putting x=Sin)

4  1.3.1  π  π
     .
3  2.4.6  2  24

Problem 3. Change the order of integration in


π / 2 2 a cosθ
I   f (r, θ )r dr dθ
0 0

Solution We have r =2a cos represents a circle with centre (a, 0) and radius a.

Since 0    /2 the region of integration is the


semicircular disc lying in the first quadrant.

In this region r varies from 0 to 2a.


Futher r = 2a cos implies  = cos-1 (r/2a).

Hence for each fixed r, varies from 0 to cos-1(r/2a).


2 a cos1 ( r / 2 a )
Hence I    f (r, θ ) r dθ dr .
0 0

Manonmaniam Sundaranar University, Directorate of Distance & Continuing Education, Tirunelveli.


52
Problem 4. Evaluate I   e y / x dxdy where D is the region bounded by the straight lines y=x;
D

y=0 and x=1.

Solution. The rigion D is a triangle as shown in the figure.

In this region x varies from 0 to 1. For each fixed x,y varies from 0 to x.
1 x
 I    e y / x dy dx
0 0

  dx
1
  xe y/x

0 0

1
  x(e  1)dx 
1
e  1 .
0
2

 x y 2 dx dy where D is the circular disc x2+y2  1.


2
Problem 5: Evaluate
D

Solution. In D, x varies from -1 to 1.

For a fixed x,y varies from  1  x 2 to 1  x 2


1 1 x 2
  x y dx dy  
2 2
x
2
y 2 dy dx
D 1  1 x 2

1 1 x 2
 4 x
2
y 2 dy dx
0 0

1 x 2
1 
1
 4  x 2 y 3  dx
0 0
3
1
 4 x 2 (1  x 2 )3 / 2 dx
0

 /2
4
  sin  cos 4 d (Putting x= sin)
2

3 0

 1.3.1  π 
 4  
 2.4.6  2 
= /24.

Manonmaniam Sundaranar University, Directorate of Distance & Continuing Education, Tirunelveli.


53
 x 
 y 2 dx dy where D is the region bounded by y=x2, x = 2 and y=1.
2
Problem 6. Evaluate
D

Solution. The region of integration is as shown in the figure.


In this region x varies from 1 to 2 and for each fixed x,y varies from 1 to x2.
2 x2
 
  x  y 2 dx dy    x 2  y 2 dy dx.
2
 
D 1 1

x2
 1 
2
   x 2 y  y 3  dx
1
3 1

 1 
2
   x 4  x 6 dx
1
3 
2
1 1  1286
  x5  x 7  
5 21 1 105


e y
Problem 7. Evaluate I    dydx.
0 x
y

Solution. We notice that we must integrate first w.r.t.x. Hence we change the order of
integration. The region of integration is as shown in the figure. (We note the D is an
unbounded region and the given integral is an improper double integral).

In the region D, y varies from 0 to . For each fixed y,x varies from 0 to y.

 y  y
e y  e y 
I    dx dy    x  dy
0  0
0 0
y y
 

  e dy   e
y
 y

0 0

= 1.
Exercises.
1. Evaluate the following integrals.

Manonmaniam Sundaranar University, Directorate of Distance & Continuing Education, Tirunelveli.


54
1 2 1 2
(a)   ( x  2) dy dx
0 0
b) I    ( x 2  y 2 ) dx dy
0 0

1.2 TRIPLE INTEGRALS:

The definition triple integrals for a funtion f(x,y,z) defind over a region D
is R3 is analogus to the definition of double integral is defind is the definition .
we replace rectangles by parallelopipeds and area by volume to obtain the
corresponding definition of triple integrals.

A triple integral of a function defind over a region D is denoted by

 f ( x, y, z) dx dy dz
D
or  f ( x, y, z) dV or  f ( x, y, z) d ( x, y, z).
D D

The triple integral can be expressed as an integrated integrals in several


ways.

For example is a region D in R3 is given by

D = {(x,y,z)/ a  x  b; 1(x)  y  2(x) 1(x,y)  z  2(x,y) then

𝑏 Φ2 x Ψ2 x,y

𝑓 𝑥, 𝑦, 𝑧 𝑑𝑥 𝑑𝑦 𝑑𝑧 = 𝑓 𝑥, 𝑦, 𝑧 𝑑𝑧 𝑑𝑦 𝑑𝑥
𝐷 𝑎 Φ1 x Ψ1 (x,y

This can also be written as


𝑏 Φ2 x Ψ2 x,y

𝑑𝑦 𝑓 𝑥, 𝑦, 𝑧 𝑑𝑧
𝑎 Φ1 x Ψ1 (x,y

Manonmaniam Sundaranar University, Directorate of Distance & Continuing Education, Tirunelveli.


55
Similarly under suitable conditions a given triple integral can be
expressed as an iterated integral in five other ways by permuting the variables.

Solved Problems.
a x y

Problem 1. Evaluate I     xyz dz dy dx


0 0 0

Solution:
y
1 
a x
I     xyz 2  dy dx
0 0 0
2
x
1 1 
a x a
1
   xy 3 dy dx    xy 4  dx
200 2 0 4 0
a
1 1 
a
1
  x 5 dx   x 6 
80 8 6 0

= a6/48

Problem 2. Evaluate I=  xyz dx dy dz where D is the region bounded by the


D

positive octant of the sphere x2+y2+z2 = a2.

Solution:
The projection of the given region in the x-y plane (z = 0) is the region
bounded by
the circle x2+y2 = a2 and lying in the first quadrant as shown in the figure.

In the given region x varies from 0 to a. For a fixed x,y varies from 0 to
(a 2  x 2 ) .

For a fixed (x,y), z varies from 0 to


(a 2  x 2  y 2 ) .

Manonmaniam Sundaranar University, Directorate of Distance & Continuing Education, Tirunelveli.


56
a ( a2  x2 ) ( a2  x2  y 2 )

I     xyz dz dy dx
0 0 0

a ( a2  x2 )
1
   xy (a
2
 x 2  y 2 )dy dx
20 0

a
1
  x(a 2  x 2 ) 2 dx (Verify)
80
a
1 1 
   (a 2  x 2 ) 3 
16  3 0

= a6/48.

loga x x  y

  e
x y  z
Problem 3. Evaluate I  dz dy dx .
0 0 0

  e 
loga x
x y  z x y
Solution. I 0 dy dx.
0 0

  e 
loga x
2 ( x y )
  e x y dy dx
0 0

log a x
 1 2( x y ) 
 
0
 2 e  e x y  dx
0
log a
 1 4x 3 2x x 
 
0
 2 e  2 e  e  dx

log a
1 3 
  e 4 x  e 2 x  e x  dx
8 4 0
1 4 3 2 3
= a  a a .
8 4 8

dx dy dz
Problem 4. Evaluate I   Where D is the region bounded by the
D
( x  y  z  1)3

planes x = 0, y = 0, z = 0 and x+y+z = 1.

Manonmaniam Sundaranar University, Directorate of Distance & Continuing Education, Tirunelveli.


57
Solution. The given region is a tetrahedron. The projection of the given region
in the x-y plane is the triangle bounded by the lines x = 0, y = 0 and x+y=1 as
the shown in the figure.

In the given region x varies from 0 to 1. For each fixed x,y varies from 0 to
1-x. For each fixed (x,y), z varies from 0 to 1-x-y.

1 1 x 1 x  y
dz dy dx
I    
0 0 0
( x  y  z  1)3

 x  y  z  1 
1 1 x
1 2 1 x  y
  0 dy dx
20 0

1 1 x
1 
  4  x  y  z  1
1
2 0
2
  dy dx
0

1 x
1 1 
1
    y  ( x  y  1) 1  dx
2 0 4 0

1 1 1 
1
1
   (1  x)   ( x  1) dx
2 0 4 2 
1
1 1 1 1 
   x  x 2  x  log( x  1)
2 4 8 2 0
1 5
 log 2  .
2 16
Excersies.
1. Evaluate the following triple integrals.
1 1 2
a)    xyz dz dy dx
0 0 x2  y 2

a b c
b)    ( x  y  z) dx dy dz
0 0 0

1 2 2

 dx  dy  x yz dz
2
c)
0 0 1

Manonmaniam Sundaranar University, Directorate of Distance & Continuing Education, Tirunelveli.


58
1.3 JACOBIANS:
In this section we introduce the concept of Jacobian of a transformation
which plays an important role in change of variables in double and triple
integrals.

Definition. Consider the transformation given by the equations

x = x(u, v, w); y = y(u, v, w); z =z (u, v, w) where the functions x,y,z have
continuous first order partial derivatives.
The Jacobian J of the transformation is defind by
x x x
u v w
y y y
J
u v w
z z z
u v w
 ( x, y, z )
The jacobian is also denoted by J 
(u, v, w)

For a transformation in two variables x = x (u,v) and y = y(u,v) the


 ( x, y )
Jacobian is given by a determinant of order two. Hence J  .
 (u, v)

Examples.

1. The transformation from cartesian coordinates (x,y) to polar coordinates (r,)

Soln:

Given that x = rcos and y = r sin.

𝜕𝑥 𝜕𝑦
= cosθ : = sinθ
𝜕𝑟 𝜕𝑟
Manonmaniam Sundaranar University, Directorate of Distance & Continuing Education, Tirunelveli.
59
𝜕𝑥 𝜕𝑦
= -r sinθ : 𝜕𝜃 = r cosθ
𝜕𝜃

x x
J  r   cos   r sin 
y y sin  r cos 
r 

= rcos2θ+r sin2θ = r(cos2θ+ sin2θ) = r.

2. The transformation from cartesian coordinates (x,y,z) to spherical polar coordinates


(r,,)

Soln: Given that


x = rsincos
y= rsinθ sinφ
z = rcos
Here 0     and 0    2.

x x x
r  
𝜕 𝑥,𝑦,𝑧 y y y
𝐽= =J 
𝜕 𝑟,𝜃,𝑧 r  
z z z
r  

𝑠𝑖𝑛𝜃𝑐𝑜𝑠𝜑 𝑟𝑐𝑜𝑠𝜃𝑐𝑜𝑠𝜑 −𝑟𝑠𝑖𝑛𝜃𝑠𝑖𝑛𝜑


= 𝑠𝑖𝑛𝜃𝑠𝑖𝑛𝜑 𝑟𝑐𝑜𝑠𝜃𝑠𝑖𝑛𝜑 𝑟𝑠𝑖𝑛𝜃𝑐𝑜𝑠𝜑
𝑐𝑜𝑠𝜃 −𝑟𝑠𝑖𝑛𝜃 0

= sinθcosφ[0+r2sin2θcosφ]-rcosθcosφ[0-rsinθcosθcosφ]

-rsinθsinφ[- rsin2θsinφ-rcos2θsinφ]

= r2 sin. (on simplification).

Manonmaniam Sundaranar University, Directorate of Distance & Continuing Education, Tirunelveli.


60
3. The transformation from cartesian coordinates (x,y,z) to Cylindrical coordinates (r,,z)

Soln:
given that x = rcos, y = rsin, z = z.

cos   r sin  0
 ( x, y , z )
J  sin  r cos  0
 (r ,  , z )
0 0 1

= cosθ(rcosθ)+rsinθ(sinθ) = r

4. Consider the transformation x +y = u, 2x-3y = v.


Soln:
Given x +y = u, 2x-3y = v.

1 1
 x  (3u  v) and y  (2u  v)
5 5
3 1
 ( x, y ) 5 5   1.
J  
 (u, v) 2 
1 5
5 5
Solved Problems.

Problem 1. If x+y+z=u; y+z=uv; z=uvw then find J.

Solution. From the given three transformations we get x = u-uv; y = uv-uww; z = uvw.

x x x
u v w
y y y
Now J 
u v w
z z z
u v w

Manonmaniam Sundaranar University, Directorate of Distance & Continuing Education, Tirunelveli.


61
1 v u 0
= v(1  w) u (1  w)  uv
vw uw uv

= (1-v)[u2v(1-w)+ u2vw]+u[v2u(1-w)+uv2w]

=(1-v)[u2v- u2v w+ u2vw]+u[v2u- v2u w+uv2w]

= u2v- u2v w+ u2vw-u2v2+u2v2w-u2v2w+v2u2-v2u2w+u2v2w


= u2 v

( x, y) 1
Problem 2. If u=x2-y2 and v = 2xy prove that 
(u, v) 4 u  v 2
2

( x, y ) u x uy 2x  2 y
Solution. Consider  
(u, v) vx vy 2y 2x

=4x2+4y2 = 4(x2+y2)............. (1)

We have (x2 + y2)2 = (x2-y2)2 + (2xy)2 = u2+v2.

 x2  y2  u 2  v2 ………..(2)
(u, v)
From (1) and (2) we get  4 u 2  v2
( x, y)
( x, y) 1
  .
(u, v) 4 u 2  v 2

Excercises.
(u, v) ( x, y )
1. Prove that .  1.
( x, y ) (u, v)

1.4 CHANGE OF VARIABLES IN DOUBLE AND TRIPLE INTEGRALS.


The evaluation of a double or a triple integral sometimes becomes easier when we
transform the given variables into new variables.

Manonmaniam Sundaranar University, Directorate of Distance & Continuing Education, Tirunelveli.


62
We state without proof the following theorem regarding change of variables in double
and triple intergrals.

Theorem 4.1.
Consider a transformation given by the equation x = x (u,v) and y = y(u,v) where x
and y have continuous first order partial derivatives. Let the region D in the x-y plane be
mapped into the region D* in the u-v plane. Further we assume that the Jacobian of the
transformation J0 for all points in D. Then  f ( x, y) dx dy   f x(u, v), y(u, v) J du dv.
D D*

Similarly for triple integrals we have

 f ( x, y, z) dx dy dz   f x(u, u, w), y(u, v, w), z(u, v, w) J du dv dw


D D*

We now proceed to evaluate some double and triple integrals by making appropriate
change of variables.

Solved Problems.
xy dx dy
Problem 1. Evaluate I   by transforming to plor coordinates Where D is the
D x2  y2

region enclosed by the circles x2+y2 = a2 and x2+y2 = 4a2 in the first quardrant.

Solution:
Put x = rcos and y = rsin
We know that J= r.
Further in the given domain D,
0    /2 and a  r  2a.

π / 2 2a
 r cos θ r sin θ 
I    
0 a
r
 r dr dθ

π/2 2a
1 
 
0
cos θ sin θ  r 3  dθ
3 0

π/2
7a 3

3  cos θ sin θ dθ
0

Manonmaniam Sundaranar University, Directorate of Distance & Continuing Education, Tirunelveli.


63
π/2
7a 3

3  sin θ d (sin θ )
0

7
 a 3 sin 2 
6
 
 /2
0

7a 3
 .
6

Problem 2. Evaluate the improper integral I   e  x dx .
2

  2   2 
Solution. I 2  II    e  x dx   e  y dy .
0  0 

   e ( x  y ) dx dy .
2 2

0 0

Put x = rcos and y = rsin. Hence J = r.


The region of integration is the first quadrant.
Hence r varies from 0 to  and  varies from 0 to /2.

  /2 

I    e r d dr 
r 2
 e r dr.
r 2
2

0 0
2 0

 
π 1 2 π  1 2
 
20 2
 e r d ( r 2 )   e r 
2 2 0

= /2 (1/2) = /4


I  .
2

1/ 2
 1  x2  y 2  ππ 
Problem 3. Prove that I     dx dy    1 .
2 
D 
1 x  y 
2
42 

Where D is the positive quadrant of the circle x2+y2=1.

Solution. Put x = rcos and y = r sin.


Manonmaniam Sundaranar University, Directorate of Distance & Continuing Education, Tirunelveli.
64
J = r.
Futher in D, 0  r  1 and 0    /2.

1/ 2
1 π/2
1 r2 
I      r dθ dr
2 
0 0 1 r 

1/ 2
π 1 r2 
1
    r dr
2 0  1  r 2 

 1  1 r2 
2 0  1  r 4
  r dr

π  1 t 
1

4 0  1  t 2
   dt (by putting r2 = t)



4
sin 1
t  (1  t 2 )1/ 2  1
0

  
   1.
42 

Problem 4. Prove that


1/ 2
 a 2b 2  b 2 x 2  a 2 y 2  ππ 
D  a 2b2  b2 x 2  a 2 y 2  dx dy  4  2  1ab
x2 y2
Where D is the Positive quadrant of the ellipse   1.
a2 b2
Solution. Put x = au and y = bv.
 ( x, y ) a 0
J    ab.
(u, v) 0 b
Let D* be the image of D under the above transformation. Then D* is the region
bounded by the unit circle u2+v2 = 1 in the first quadrant.

1/ 2
1  ( x 2 / a 2 )  ( y 2 / b 2 ) 
Now, I   
1  ( x 2 / a 2 )  ( y 2 / b 2 ) 
dx dy
D 

Manonmaniam Sundaranar University, Directorate of Distance & Continuing Education, Tirunelveli.


65
1/ 2
1  u 2  v 2 
  
1  u 2  v 2 
J du dv
D 

1/ 2
1  u 2  v 2 
 ab  
1  u 2  v 2 
du dv
D 

 
 ab ( 2  1). (by problem 3).
4

Problem 5. Evaluate 
D
x  y dx dy where D is the parallelogram bounded by the lines

x+y = 0; x+y = 1; 2x-3y = 0 and 2x-3y = 4.

Solution.
Put x+y = u and 2x-3y = v.

Then J = -1/5 (using by above example 1.3 of 4)

Also D is transformed into the rectangle bounded by the lines u = 0; u = 1; v = 0 and


V = 4.

 1
1 4 1
  u   dv du    u v0 du
1 4

0 0  5  5 0

1
4 2 
   u 3/ 2 
5 3 0
= -8/15

Problem 6. Evaluate I   xyz dx dy dz where D is the positive octant of the ellipsoid
D

x2 y2 z 2
   1.
a2 b2 c2
Solution.

Put x = au, y = bv and z = cw.

Manonmaniam Sundaranar University, Directorate of Distance & Continuing Education, Tirunelveli.


66
a 0 0
 ( x, y , z )
J   0 b 0  abc.
 (u, v, w)
0 0 c

Let D* be the image of D under the above Transformation. The D* is the positive
octant of the sphere u2+v2+w2 = 1.

 I   abc uvw du du dw


D*

=a2b2c2 
D*
uvw du dv dw.

Now, put u = r sin cos


v = r sin sin
w = r cos
Then J = r2 sin. (above example 1.3 of 2)

1 π/2 π/2

. ̇. 𝐼 = 𝑎2 𝑏 2 𝑐 2 𝑟 5 𝑠𝑖𝑛3 𝜃 𝑐𝑜𝑠𝜃 𝑐𝑜𝑠𝜑 𝑠𝑖𝑛𝜑 𝑑𝜑 𝑑𝜃 𝑑𝑟


0 0 0
1 π/2 π/2

= 𝑎2 𝑏 2 𝑐 2 𝑟 5 𝑑𝑟 𝑠𝑖𝑛3 𝜃 𝑐𝑜𝑠𝜃 𝑑𝜃 𝑠𝑖𝑛𝜑 𝑐𝑜𝑠𝜑 𝑑𝜑.


0 0 0

1 6 1 1 𝜋/2 1 𝜋/2
= 𝑎2 𝑏2 𝑐 2 𝑟 𝑠𝑖𝑛 4
𝜃 𝑠𝑖𝑛 2
𝜑
6 0 4 0 2 0

𝑎2𝑏2𝑐2
= .
48

Excercise 1. Evaluate the following double integrals using change of variables or otherwise

over the region indicated.

a). 
D
( x 2  y 2 ) dx dy ;D is the a region bounded by the circle x2 + y2 = a2.

Manonmaniam Sundaranar University, Directorate of Distance & Continuing Education, Tirunelveli.


67
 e
 ( x2  y 2 )
b). dx dy D is the region bounded by the circle x2 + y2 = a2.
D

c).  ( x 2  y 2 ) dx dy D is the region in the x-y plane bounded by x2 + y2 = 4 and


D

x2+y2 = 9.

Manonmaniam Sundaranar University, Directorate of Distance & Continuing Education, Tirunelveli.


68
UNIT – 3 : FIRST ORDER DIFFERENTIAL
First order differential: equations of higher degree- solvable for p, x and y- Clairaut’s form/
linear differential equations of second order- Particular integrals for functions of the form,
Xn, eax, eax(f(x)). Second order differential equations with variable coefficients.

1.1 Equations of the first order, but of higher degree.

dy
TYPE A:- Equations solvable for p ( ).
dx
dy
We shall denote hereafter by p.
dx

Let the equation of the first order and of the nth degree in p be
pn + P1pn-1+P2pn-2+….+ Pn=0 …(1)

where P1, P2……Pn denote functions of x and y.

Suppose the first number of (1) can be resolved into factors of the first degree of the
form

(p - R1) (p - R2) (p - R3) …. (p - Rn)

Any relation between x and y which makes any of these factors vanish is a solution of (1).

Let the primitives of p - R1= 0, p - R2= 0, etc be

1(x, y, c1) = 0, 2 (x, y, c2) = 0 … n (x, y, cn) = 0.

respectively, where c1, c2, …… cn are arbitrary constants. Without any loss of
generality, we can replace c1, c2, ….cn by c, where c is an arbitrary constant. Hence the
solution of (1) is

1(x, y, c).2 (x, y, c) … n (x, y, c) = 0.

Manonmaniam Sundaranar University, Directorate of Distance & Continuing Education, Tirunelveli.


69
Examples.

1) Solve x2p2 + 3xyp + 2y2 = 0.


Soln:
y 2y
Solving for p, p =  or   . (Quadratic eqn)
x x
dy y dy y
 ;  0
dx x dx x
𝑑𝑥 𝑑𝑦
+ =0
𝑥 𝑦

Integrating,

log x+log y =log c

Therefore xy = c … (1)

dy 2 y dy 2 y
 ;  0
dx x dx x
dy 2dx
 0
y x
Integrating,

log y+2logx = log c

ie) log y + log x2 = log c

yx2 = c …(2)

The solution is (xy – c) (yx2-c) = 0.

 2y  y2 y2
2) Solve p   x  y   p  xy  2  y 
2
 0.
 x  x x

Manonmaniam Sundaranar University, Directorate of Distance & Continuing Education, Tirunelveli.


70
Soln:
y y
Solving for p, p   y (or ) p   x (Quadratic eqn)
x x

dy  y  dy y
   y  (or )   x
dx  x  dx x

dy  1  dy y
   1dx or    x
y x  dx x

Integrating on,
log y = log x –x+logc

ie)logy-log x = -x+log c
y
log = -x + log c
x
𝑦
=ce-x
𝑥

i.e., y = cxe-x

dx dx
   
The second equation is linear in y. Hence the solution is ye x
  xe x
dx  c

y
i.e.,  x  c
x
i.e., y = - x2 + cx

The general solution is (y-cxe-x) (y+x2-cx) = 0.

TYPE B :-Let the differential equation (1) of 2 be put in the form f (x, y, p) = 0. When it
cannot be resolved into rational linear factors as in 5.1, it may be either solved for y or x.

Equations solvable for y.

f(x,y,p) = 0 can be put in the form

Manonmaniam Sundaranar University, Directorate of Distance & Continuing Education, Tirunelveli.


71
y = F (x,p) ….(1)

𝑑𝑝
Differentiating with respect to x, p = φ (x,p,𝑑𝑥 )

This, being an equation in the two variables p and x, can be integrated by any of the
foregoing methods. Hence we obtain

Ψ(x,p,c) = 0 ….(2)
Eliminating p between (1) and (2), the solution is got.

Equations solvable for x:


f (x, y,) = 0 can be put in the form
x= F (y, ) ….(1)

1  dp 
Differentiating with respect to y,    y, p, .
  dy 

Integrating leads to Ψ (y, p, c) = 0 ….(2)

Eliminating  between (1) and (2), the solution of (1) is got.

Examples

1) Solve xp2-2yp+x=0:

Soln:
( p 2  1)
Solving for y, y  x
2p
𝑑𝑦
Differentiating with respect to x, [𝑑𝑥 = 𝑝]

p2  1 p 2  1 dp
p x
2p 2 p 2 dx

Manonmaniam Sundaranar University, Directorate of Distance & Continuing Education, Tirunelveli.


72
p2  1 dp ( p 2  1)
x 
2p dx 2 p2

dx dp
 
x p

Integrating, p = cx.

Eliminating p between this and the given equation, the solution is

2 cy = c2x2+1.

2) Solve x = y2 + log p … (1)

Soln:
(This is easily solvable for x only)

Diffenentiating with respect to y,


1 1dp
 2y 
p pdy

dp
 2 py  1. This is linear in p and hence.
dy

p e y   e y dy  c.
2 3
….(2)

(It must be noted that the integral on the R.H.S. cannot be integrated in finite terms.)

The eliminant of p between (1) and (2) gives the solution.

Note. In the above problem, the solution has not been got explicity by eliminating p.
But we have x and y expressed in terms parameter p. This will do.
Manonmaniam Sundaranar University, Directorate of Distance & Continuing Education, Tirunelveli.
73
1.2 Clairaut’s form.

The equation known as Clairaut’s is of the form


y= px + f (p) ….. (1)

Differentiating with respect to x, p = p + x  f ( p)


dp
dx
dp
 0 or x + f (p) = 0.
dx

dp
 0 , integrating on, p = c, a constant.
dx

The solution of (1) is y = cx + f (c).

We have to replace p in Clairaut’s equation by c. The other factor y + f (p) = 0 taken along
with (1) give, on eliminating of p, a solution of (1). But this solution is not included in the
general solution (2). Such a solution as this is called a singular solution.

Examples.

1) Solve y = (x-a) p-p2


Soln:

This is Clairaut’s equation; hence the solution is

y= (x-a) c- c2

2) Solve y = 2px + y2p2


Soln:

Putting X =2x and Y =y2,

dX=2dx: dY = 2ydy

Manonmaniam Sundaranar University, Directorate of Distance & Continuing Education, Tirunelveli.


74
dY
P   yp
dX

The equation transforms into Y = XP + P2

This is Clairaut’s equation; hence Y=cX+c2.

The solution is y2 = 2xc + c.

We have an extended form of Clairaut’s equation of the type


y = x f (p) +  (p).

Differentiating with respect to x … (1)

dp
p = f (p) + [xf '(p) + ' (p)⦌
dx
dx xf ' ( p)  ' ( p)
 
dp f ( p)  p p  f ( p)

This is linear in X and hence gives F (x, p , c ) = 0

The eliminant of P between this equation and (1) give the solution of (1).

Example.
1) Solve y = xp+x(1+p2)1/2
Soln:
Given y = xp+x(1+p2)1/2

Differentiating with respect to x.

dp  xp 
p  p  (1  p 2) ) 2  x  
1

dx  1  p 2 

Manonmaniam Sundaranar University, Directorate of Distance & Continuing Education, Tirunelveli.


75
1  p 2  p dx
Hence dp   0.
(1  p 2 ) x

dp p dp dx
Integrating,  1  p2

1 p 2
   log c
x

1
i.e., log ( p  1  p 2 )  log(1  p 2 )  log x  log c.
2

log ( p  1  p 2  1  p 2 ) x  log c

p 1  p 2

 1  p2 x  c …(1)

Eliminating p between (1) and (2) the solution is got.

1.3 LINEAR DIFFERENTIAL EQUATIONS WITH CONSTANT


COEFFICIENTS

A typical linear equation of the second order with constant coefficients is


d 2y dy
a 2
 b  cy  X …… (1)
dx dx

where a,b ,c are constants and X is a function of x.

Let us consider (1) without the second number,

d2y dy
i.e., a 2
 b  cy  0 …..(2)
dx dx

The solution of this equation (2) is called the complementary function of (1).

Manonmaniam Sundaranar University, Directorate of Distance & Continuing Education, Tirunelveli.


76
𝑑𝑦
To solve (2), assume as a trial solution y = emx for some value of m. Now =
𝑑𝑥
𝑑2𝑦
𝑚 𝑒 𝑚𝑥 𝑎𝑛𝑑 𝑑𝑥 2 = 𝑚2 𝑒 𝑚𝑥 . Substituting these values in (2), we get

emx(am2+bm+c = 0 ….. (3)

Hence m satisfies am2+bm+c=0. This equation in m is called the auxiliary equation.

Three cases can arise in the solution of the auxiliary equation.

Case(i). Let the auxiliary equation (3) has two real and distinct roots m1 and m2.

 y=em1x and y = em2x are solutions of (2).

Hence A em1x, B em2x are solutions of (2), where A and B are arbitrary constants. Thus
y= A em1x + B em2x is the most general solution of (2) as the number of constants occuring in
this solution is two, equal to the order of the differential equation.

Case (ii). Let the auxiliary equation (3) has two roots equal and real.

Let m2=m1. The solution y= A em1x + B em2x becomes.

(A+B) em1x = c em1x ……4

where c is a single arbitrary constant equal to A+B. Thus the number of constants is reduced
to one which is one short of the order of the differential equation (2) and therefore (4) ceases
to represent the general solution. Hence we proceed as follows :

Let us put m2 = m1 + ϵ and allow ϵ to tend to zero.

The solution is
y = A em1x + B e(m1+ϵ) x
= em1x + (A + B eϵ x)

Manonmaniam Sundaranar University, Directorate of Distance & Continuing Education, Tirunelveli.


77
  2 x 2 
= e m1x  A  B1  x   .....
  2 
by the exponential theorem
= em1x (A+B+ϵ B x) the other terms tending to zero as ϵ ⟶ 0.
We can choose B sufficiently big so as to make ϵ B finite asϵ ⟶ 0.
and A large with opposite sign to B so that A + B is finite.

If A + B + C and ϵB = D, the solution corresponding to two equal roots m1 is em1x (C


+ D x).

Case (iii). Let the auxiliary equation has imaginary roots.

As imaginary roots ocuur in pairs, let m1 = + i where  and  are real; then m2 = 
- i.

The solution is y = A e (+i)x + B e-i)x


= e x [A eix + B-ix]
= e x {A cos  x + i sin  x + B (cos x – isin x)} by Euer’s
formula.

= ex (c cos  x + Dsin x), where C and D are arbitrary


constants.

This can also be written as y = A e x cos (x+B), where A and B arbitrary constants.

Examples.

d2y dy
1) Solve 2
 5  4y  0
dx dx
Soln:
The auxiliary equation is m2 - 5m + 4 = 0

Manonmaniam Sundaranar University, Directorate of Distance & Continuing Education, Tirunelveli.


78
( m -1) (m-4)=0

m=1 and m=4.

complimentary function y = A ex + B e4x

d2y
2)Solve  9y  0
dx 2
Soln:
The auxiliary equation is m2 -9= 0

m2-32 = 0 ; (m-3)(m+3)=0

m=3 and m=-3


C.F y = A e3x + B e3x

d2y dy
3)Solve 2
2  y0
dx dx
Soln:
The auxiliary equation is m2 + 2m + 1 = 0, i.e., (m+1)2 = 0.
m = - 1 twice.
C.F y = e-x (A + B x).

d2y dy
4)Solve 2
4 40
dx dx
Soln:
The auxiliary equation is m2 + 4m + 22 = 0, i.e., (m+2)2 = 0.
m = - 2 twice.
 y = e-2x (A + B x).

Manonmaniam Sundaranar University, Directorate of Distance & Continuing Education, Tirunelveli.


79
d2 dy
5) Solve 2
 3  5y  0
dx dx
Soln:
The auxiliary equation is m2 – 3m + 5 = 0.
3  11i
Solving,(quadratic eqn form) m 
2

3x 
  11   11  
 y  e 2  ASin  x   B sin x 

  2   2  
d2y dy
6)Solve 2
 4  13 y  0
dx dx
Soln:
The auxiliary equation is m2 +4m 13 = 0.
 4  6i
Solving,(quadratic eqn form) m 
2
m  2  3i

 y  e2 x ASin 3x   B sin3x 

Exercises:

Solve the following equations :-

d2y dy d 2 y dy
1. 2
 6  8y  0 2.   4y  0
dx dx dx 2 dx

d2y dy d2y dy
3. 2
 4  4y  0 4. 2
 3  8y  0
dx dx dx dx

The operators D and D-1


d 2 d2
Let D stand for the operator and D for
dx dx 2
This symbol D satisfies the commutative, associative, and distributive laws; for
(Dm+Dn) u = (Dn+Dm)u = Dnu +Dm u

Manonmaniam Sundaranar University, Directorate of Distance & Continuing Education, Tirunelveli.


80
Dm.Dn u = Dn.Dm u = Dm+nu
and D ( u+v) = D (v+u).

We can define the inverse operator D-1 as one such that when it operates on any
function of x and subsequently the operation by D is performed, the function is left unaltered.
Thus D-1 represents integration.

1 1
We shall define the operator as the inverse of the operator f (D). i.e., X
f ( D) f ( D)
is that function of x which, when operated upon by f(D) yields X.

1
We shall assume that the order of the operators f(D) and can be interchanged.
f ( D)

 1  1
The f (D)  x  f ( D) x  x
 f ( D)  f ( D)

1.4 Particular integral.

Consider equation (1) which can be written symbolically as

(aD2+ bD + c) y = x

or shortly f (D) y = X, where f (D) = a D2 + b D + c .

Let y = u be a particular solution of this equation.

Let Y be the complementary function of (1)

Then y = Y + u is the general solution of (1).


u is called the particular integral of (1).
1
In symbolic form, it is written as X
f ( D)

Manonmaniam Sundaranar University, Directorate of Distance & Continuing Education, Tirunelveli.


81
1
i.e., P.I  X
aD  bD  c
2

Special methods of finding P.I.

(a) Let X be of the form ex

D ex =  ex

More generally, Dn ex= (n) ex

 f ( D)e x  f ( ) e x as f ( D) is a quadratic in D in our case.

1 1 x
Operation on both sides by , e x
 f ( ) e
f ( D) f ( D)

1 x 1 x
If f ()  0, e  e
f ( D) f ( )

Case (i). Hence the rule is:

1
In ex , replace D by  if f ()  0
f ( D),

Case (ii) If f () = 0,  satisfies the auxiliary equation f (m) = 0. Then we proceed as follows:

(i) Let the auxiliary equation have two distinct roots m1 and m2 and let  = m1.

Then f (m) =a (m- m1) (m - m2)


= a (m-) (m - m2)
1
P.I. = ex
a( D   )( D  m2 )

Manonmaniam Sundaranar University, Directorate of Distance & Continuing Education, Tirunelveli.


82
1 1
= ex bycase (i)above
a( D   ) (  m2 )
1
To find ex ,let us put z =
(D   )
𝑑𝑧
Operating on both sides by D – α,𝑑𝑥 – α z = eαx.

This is linear equation of the first order; hence


z e-αx = ʃ e-αx dx = x.

(It must be noted that no constant of integration is added as we are evaluating only a

particular integral.

If the constant be added there will occur in the general solution 3 constants as there

are already in the C.F. and thus one constant will be too many).

.˙. Z = x eαx

1
Hence P.I. = xe x .
a(  m2 )

(ii) Let the auxiliary equation have two equal rootes each equal to α.

i.e., m2 = m1 = α.

.˙.f (m) = a (m-α)2

1 1
P.I .  ex  ex
a( D   ) 2
a( D   )( D   )

1 𝑋 𝑒 𝛼𝑥
=𝑎 𝐷−𝛼

1 dz
If z  xe αx ,  αz  xe αx
Dα dx

Manonmaniam Sundaranar University, Directorate of Distance & Continuing Education, Tirunelveli.


83
x2
Sloving, zeαx   xdx  (no constant is added).
2
x 2 αx
z  e
2
x 2 αx
 P.I .  e
2

Examples.

Ex.1.Slove (D2+5D+6)y = ex.

Soln:

To find the C.F of (D2+5D+6)y = 0.

The auxillary equation is m2 + 5m + 6 = 0.

(m+2)(m+3)=0

m = -2 and -3.

C.F.= A e-2x + B e-3x.


1
P.I .  ex
D  5D  6
2

1 x
 e on replacing D by 1
12

ex
y  Ae 2 x  Be 3 x 
12

2) Slove (3D2+D -14)y = 13 e2x.

Soln:

To find the C.F of (3D2+D-14)y =0.

The Auxillary equation is 3m2 + m - 14 = 0.

Sloving, m = 2 and -7/3

.˙. y = A e-2x + B e-7x/3


1
P.I .  13e 2 X
( D  2)(3D  7)

Manonmaniam Sundaranar University, Directorate of Distance & Continuing Education, Tirunelveli.


84
13 1
 e2 x
13 D  2

1
 e 2 x  xe 2 x by 4 case (ii)
D2

.˙. y = Ae2x + Be-7x/3 + xe2x

3) Slove (D2- 2m D + m2)y = emx.

Soln:

To find the C.F of (D2 -2mD + m2) = 0.

The auxillary equation is K2 - 2mk + m2 = 0.

(Note. K is used here instead of the usual m as there is already another m).

i.e., (k -m)2 = 0

.˙. k = m twice

C.F. = emx (A + Bx).


1 x 2 mx
P.I .  e mx
 e by 4 case (ii)
( D  m) 2 2

 x2 
 y  e mx  A  Bx  
 2 

Excersice
Solve the following equations:-
1. (D2 - 5D + 6) y = e4x. 2. (D2 - 6D + 13) y = 5e2x.
3. (D2 - 4D + 6) y = 5e-2x. 4. (D2 - 2D + 1) y = 2e3x.

(b) Let X be of the form cosαx or sin αx, where α is a constant.

D sin αx = cosαx.

D2 sin αx = -α2 sin αx.

 (D2) sin αx= (-α2) sin dx, as  (D2) is a rational integral function of D2.

Manonmaniam Sundaranar University, Directorate of Distance & Continuing Education, Tirunelveli.


85
1 𝜙 (−𝛼 2 )
Operating on both sides by 𝜙 (𝐷 2 ) , sin 𝛼𝑥 = sin 𝛼𝑥.
𝜙 (𝐷 2 )
1 1
Case (i). If 𝜙 −𝛼 2 ≠ 0, 𝜙 sin 𝛼𝑥 = sin 𝛼𝑥
𝐷2 𝜙 −𝛼 2

Hence the rule is :

Replace D2 by -α2, Provided (-α2) ≠ 0.

The same rule applies if sin αx be replaced by cosα x


1 1
i.e.,𝜙 cos 𝛼𝑥 = cos 𝛼𝑥.
𝐷2 𝜙 −𝛼 2

Case (ii).If (-α2) = 0,D2 +α2 is a factor of (-D2).

1
To evaluate sin x, the above rule fails. Hence the following procedure is adopted.
D  2
2

1 1
sin x  2 . Imaginary part of eαix as
D 
2 2
D  2

eiαx = cos αx + i sin αx by Euler’s formula;


1
= imaginary part of e iαx
D α
2 2

1
= " eiαx
( D  αi )( D  αi )

1
= " eiαx by 4 (a)
( D  αi) 2αi

xe ix
= " by 4(a)
2i

xi
= "  (cos x  i sin x)
2

x cos x
=
2

Manonmaniam Sundaranar University, Directorate of Distance & Continuing Education, Tirunelveli.


86
1 x sin ax
Similarly, cos ax  .
D a
2 2
2a

Examples.

Ex. 1. Solve (D2 - 3D +2) y = sin 3x.


Soln:
To find the C.F of (D2 - 3D + 2) y = 0.
The auxiliary equation is m2 - 3m + 2 = 0.
(m-2)(m-1)=0
m = 1 and 2.
C.F. = A ex + B e2x.
1
P.I .  sin 3x
D  3D  2
2

1
 sin 3x on replacing D2 by -9 by 4(b)
 9  3D  2
1
 sin 3x
3D  7
In order to apply the above rule, we must aim at getting D2 terms only in the
denominator; hence we write
1 3D  7 3D  7
= 
3D  7 (3D  7)(3D  7) 9 D 2  49
and proceed.
 (3D  7)
P.I .  sin 3x.
9 D 2  49
d
3 (sin 3x)  7 sin 3x
 dx by 4(b)
9(9)  49
 9 cos 3x  7 sin 3x

 130
y = C.F. + P.I.

Manonmaniam Sundaranar University, Directorate of Distance & Continuing Education, Tirunelveli.


87
d2y
Ex. 2 Show that the solution of the differential equation  4 y  A sin pt which is such
dt 2
1
(sin pt  p sin 2t )
dy 2
that y = o and  0 when t = 0, is y  A if p ≠2.If p =2, show that
dt 4  p2
A(sin 2t  2t cos 2t )
y
8
Soln:
d
Let D stand for here.
dt
To find the C.F of (D2+4) y = 0.
The auxillary equation is m2 + 4 = 0.
m2=-4
m =  2i.
.˙. C.F. = λ cos 2t + μ sin 2t , where λ and μ are arbitrary constants.
(Note that the independent variable is t.)
1
P.I .  a sin pt
D 42

1
 A sin pt if p2≠ 4 by 4(b).
 p2  4
.˙. y = λ cos2t+ μ sin 2t+(A/4-p2) sin pt
To determine the values of λ and μ, we note that when
dy
t = 0, y = 0 and  0.
dt
.˙. 0 = λ
𝑑𝑦 𝐴𝑝
= -2λ sin2t+ 2μcos 2t+ 4−𝑃 2 cos pt.
𝑑𝑡

Ap
0  2μ 
4  p2
Ap
  .... (2)
2(4  p 2 )

1
A(sin pt  sin 2t )
Hence, y  2
(4  p 2 )
1
If p = 2, P.I .  A sin 2t
D 42

Manonmaniam Sundaranar University, Directorate of Distance & Continuing Education, Tirunelveli.


88
Ap 2it
= Imaginary part of e
D2  4
A
= Imaginary part of e 2it
( D  2i)( D  2i )
A 2it
= Imaginary part of te
4i
 At cos 2t
=
4
At
y  λ cos 2t  μ sin 2t  cos 2t .
4
When t = 0, y = 0 .˙. λ = 0.
dy A
 2 sin 2t  2 cos 2t  (cos 2t  2t sin 2t )
dt 4

𝐴 𝐴
0 = 2μ - , μ = .
4 8

A(sin 2t  2t cos 2t )
y .
8

3)Slove (D2 - 4D + 3)y = sin 3x cos 2x.


Soln:
To find the C.F of (D2 - 4D +3)y = 0.
To auxillary equation is m2 - 4m + 3 = 0.
(m-3)(m-1)=0
m =1 and 3.
C.F. = A ex + B e3x
1
P..I .  sin 3x cos 2 x
D  4D  3
2

1 sin 5 x  sin x

D  4D  3
2
2
1 sin 5 x 1 sin x
 
 25  4 D  3 2  1  4D  3 2

Manonmaniam Sundaranar University, Directorate of Distance & Continuing Education, Tirunelveli.


89
2 D  11 1  2D
 sin 5 x  sin x
 4(4 D 121)
2
4(1  4 D 2 )

10 cos 5x  11sin 5x sin x  2 cos x


 
884 20
y = C.F. + P.I.
10 cos 5 x  11sin 5 x sin x  2 cos x

y =A ex + B e3x + 884 20

4) Slove (D2 + 16) y = 2e-3X + cos4x.

Soln:
To find the C.F of (D2 + 16 )y = 0.
The auxiliary equation is m2 + 16 = 0.
m2+42=0 ; m = 4i.
C.F. is = Acos 4x + B sin 4x.
2
Now 2e3 x  e 3 x
D  16
2

= (2/7) e-3x
1
P.I.2 corresponding to cos 4 x  cos 4 x
D  16
2

1
 Real part of e4ix
D  16
2

1
= Real part of e 4ix
( D  4i)( D  4i)
1 4ix
= Real part of xe
8i
xi
= Real part of  (cos 4 x  i sin 4 x)
8
x
= sin 4 x .
8
2 x
 y  A cos 4 x  B sin 4 x  e 3 x  sin 4 x .
7 8
Manonmaniam Sundaranar University, Directorate of Distance & Continuing Education, Tirunelveli.
90
Excersices

Solve the following equations:-


1. (D2 + 4) y = sin 3x. 2. (D2 + D +1)y = sin 2x.
3. (D2-8D+9)y = 8cos5x. 4. (D2-2D-8)y = 4 cos 2x.

(c) Let X be of the form xm (a power of x), m being a positive integer.

1
To evaluate x m , raise f(D) to power -1 and expand in ascending powers of D
f ( D)
as far as Dm. (The higher powers of D operating on Xm give Zero and hence are omitted.)
Thesse terms in the expansion of f{(D)}-1 operating on xm give the particular integral
required.

Examples.

1)Solve (D2 +D +1) y = x2.


Soln:
To find the C.F of (D2+D +1) y = 0:
The auxillary equation is m2 + m + 1 = 0.
 1  3i
Solving, m 
2

 3x 3x 
C.F .  e  x / 2  A cos  B sin .

 2 2 
1
P.I .  x2
1 D  D 2

1
.  (1  D  D2 ) x 2

 {1  ( D  D2 )  ( D  D2 )2}x2 , the powers of D higher than 2 are


dropped.
 (1  D) x 2  x 2  2 x .

y = C.F.+ P.L.

Manonmaniam Sundaranar University, Directorate of Distance & Continuing Education, Tirunelveli.


91
 3x 3x 
y  e x / 2  A cos  B sin   x2  2 x

 2 2 
2) Solve (D2+4D+5)y = ex+x3+cos2x.
Soln:
To find the C.F of (D2+4D+5)y = 0.
The auxiliary equation is (m2+4m+5) = 0.
Solving, m = -2i.
C.F. = e-2x(A cos x + B sinx).
1
P.I. Corresponding to ex  ex
D  4D  5
2

1 1
 ex  ex
1 4  5 10
1
P.I. Corresponding to x3  x3
5  4D  D2
1
1  4D  D2  3
 1   x
5 5 

1   4D  D2   4D  D2   4D  D2  
2 3

 1           x3
5  5   5   5  

1  4 D 11D 2 24 3  3
 1    D x
5 5 25 125 

1  3 12 x 2 66 x 144 
 x    
5 5 25 125 

P.I.2 can also be found by assuming

y = A x3 + B x2+C x +D.
1
P.I.3 corresponding to cos 2 x  cos 2 x
D  4D  5
2

1
 cos 2 x on putting -4 for D2
1  4D

1  4D cos 2 x  8 sin 2 x
 cos 2 x 
1  16 D 65
Manonmaniam Sundaranar University, Directorate of Distance & Continuing Education, Tirunelveli.
92
y = C.F. + P.I.1 + P.I.2+ P.I.3

1 x 1  3 12 x 2 66 x 144  cos 2 x  8 sin 2 x


y = e-2x(A cos x + B sinx)  e + x    
10 5 5 25 125  65
Exercise:
Solve the following equations : -
1. (D2-1) y = 2 + 5x.
2. (D - 1)2 y = x.
3. (D2 +D +1) y = x + sin x.

(d) X is of the form eax V, where V is any function of x.

D2(eax V) = a eax V+ eaxDv = eax (D+a) V.

D2(eax V) = D{eax(D+a) V}
= aeax (D + a) V + eax (D2 + aD) V
=eax(D2 + 2aD + a2)V = eax (D+a)2 V.
It follows by induction that Dn(eax V) = eax(D+a)n V.
f (D) eaxV = eax f (D+a) V.
1
Operating on both sides by
f ( D)
1
eaxV  eax f ( D  a)V .
f ( D)
If we set f(D+a) V= V1, then this result gives,
1 1 ax
eax V1  e V1
f ( D  a) f ( D)
1 1
Hence eax X  eax X.
f ( D) f ( D  a)

Examples.
Slove (D2 -4D +3) y = e-xsinx.
Soln:
To find the C.F of Slove (D2-4D+3)y = 0.

Manonmaniam Sundaranar University, Directorate of Distance & Continuing Education, Tirunelveli.


93
The auxiilary equation is m2 -4m + 3 = 0.
(m-1)(m-3)=0
.˙. m = 1 and 3
C.F. = A ex + B e3x.
1
P.I .  e x Sinx
D  4D  3
2

1
 ex Sinx by the above rule
( D  1)  4( D  1)  3
2

1
 ex Sinx
D  6D  8
2

1
 ex sin x on putting -1 for D2
7  6D
7  6D
 ex Sinx
49  36 D 2
7 sin x  6 cos x
 ex
85
y = C.F. + P.I.
7 sin x  6 cos x
y=A ex + B e3x+ e x
85

Solve (D2+2D+5)y = xex.


Soln:
To find the C.F of (D2+2D+5) y = 0.
The auxiliary equation is m2 + 2m + 5 = 0.
Solving, m = -12.
C.F. = e-x (A cos2x + B sin 2x).
1
P.I .  xe x
D  2D  5
2

1
 ex x
( D  1)  2( D  1)  5
2

1
 ex x
D  4D  8
2

1
 ex x (as D2 can be omitted in thr dinomiator for only x occurs
8  4D
in numerator)
Manonmaniam Sundaranar University, Directorate of Distance & Continuing Education, Tirunelveli.
94
ex  D 
 1   x
8 2

ex  1
 x 
9 2
y = C.F. + P.I.
ex  1
y = e-x (A cos2x + B sin 2x)  x 
9 2
Exercises
Solve the following equations : -
1. (D2 + 1) y = (x2 + 1) ex.
2. (D2+4) y = x e2x.
3. (D2 - 4D + 3) y = excos 2x.
4. (D2 - 2D + 2)y = excosx.

1.5 Linear equations with variable coefficients.


We shall first consider the homogeneous linear equartion. A homogeneous linear
equation of the second order is of the form
d2y dx
ax 2
2
 bx  cy  X
dx dy
Where a, b, c are constrants and X is a function of X.

Method 1 By putting z = logx or x = ez, this equation can be transformed into one with
constant coeffieients.
d
We introduce here an operator θ  x . Now,
dx
dy dy dz 1 d dy dy d
  ;x   D y if D stands for
dx dz dx x dz dx dz dz
d 2 y 1 d 2 z dz 1 dy 1 d 2 y dy D
2
 2
 2  2 ( 2  )  2 ( D  1) y
dx x dx dx x dz x dz dz x
d2y
 x2  D( D  1) y
dx 2
d dy
We note that D   x θ
dz dx

Manonmaniam Sundaranar University, Directorate of Distance & Continuing Education, Tirunelveli.


95
So, putting x = ez in (1) the equation (1) becomes {a D(D-1)+ bD + c } y = Z ...... (2)
where Z is a functiuon of z into which X has been transformed. This equation (2) is a linear
equation with constant coefficients and hence the foregoing method can be adopted.

Method 2. Without transforming (1) into a linear equation with constant coeffieients, an
independent method may be given.
To find the complementary function of (1), we have to slove
d2y dy
ax 2 2
 bx  cy  0
dx dx
If xm, for some value of m, be taken as a tentative solution, then, on subsitution, we
get
am (m-1) + bm + c = 0.
This, being an equation of the seocnd degree in m, has two roots m1, m2. Hence the
complementry function of (1) is C1 xm1+ C2xm2, taking the two roots to be distinct.
If however, a root m1 be repeated twice putting m2 = m1 + є where є → 0, the
corresponding of the C.F. is
x m1 (C1  C2 x )  x m1 (C1  C2 elog x )

 2 (log x) 2 
 x m1 C1  C2 (1  . log x  etc., )
 2 
є2 being neglected as є→ 0. Putting C2є = B and C1 +C2 = A, the part of the C.F.
arising from the two equal roots m1, is x m1 ( A  B log x)

To find the Particular Integral.


d2y dy
The P.I. of ax 2
2
 bx  cy  X ............ (1) is now found.
dx dx
d
Using   the first member of (1) can be symbolically written as f(θ)y, where f(θ)
dx
= aθ(θ-1) + bθ +c.
(1) can be written as f(θ) y = X.

1 1
The P.I. is X , Where is the inverse operator defined as in 3
f (θ ) f (θ )
If f(θ) = (θ - α1) (θ - α2), the P.I. can be put either as
Manonmaniam Sundaranar University, Directorate of Distance & Continuing Education, Tirunelveli.
96
1 1
X
(θ  α1 ) (θ  α2 )

 A A2 
Or  1   X
 θ  α1 θ  α 2 
by the method of partial fractions.
It must be noted that in the first form, the order of the operators is not commutative.
Here, the operations indicated by the factors are to be taken in succession, beginning with the
first on the right. Thus the general method og finding the P.I. ultimately depends on the
1
evaluation of X.
θ α
1
To find X.
θ α
1
Let u  X.
θ α
du
By definition of inverse operator, x  u  x
dy
𝑑𝑢 𝛼 𝑋
𝑖. 𝑒. , 𝑑𝑥 − 𝑥 𝑢 = 𝑥 .

This equation is linear in u and hence its solution is

u x α   x α 1 X dx

no constant being added as this is a particular integral.

 u  x  x  1 X dx

(It is advisable for the student to commit this result to memory).

Special method of evaluating the P.I. when X is of the form xm.

d m
 xm  x (x )  m xm.
dx

d
 2 xm  x (m x m )  m 2 x m
dx

Generally, f (𝜽) xm = f (m) xm.


Manonmaniam Sundaranar University, Directorate of Distance & Continuing Education, Tirunelveli.
97
1 1
Operating on both sides by , x m  f (m) xm.
f ( ) f ( )
1 1
If, f (m)  0, xm  xm.
f ( ) f (m)
1 1
If, however, f(m) = 0, then f(𝜽) = (𝜽-m)  (𝜽) when  (m) ≠ O.P.I. becomes xm
 ( m)   m

1
 x m  x m1 x m dx by the above general method
 (m)

1
 x m log x.
 (m)

If m be repeated two times in f(m) = 0,

x m (log x)
the P.I. is , where f(m) = (𝜽-m)2.
2!

Examples:
d2y dy
Ex.1. Solve 3x 2 2
x y  x
dx dx
Soln:
d
Putting z = log x and D  the equation becomes
dz

[3D (D-1) + D+1] = ez.

The auxiliary equation is 3m2 – 2m + 1 = 0

m 1 2 i

C.F.  e z ( A cos 2 z  B sin 2 z)

Manonmaniam Sundaranar University, Directorate of Distance & Continuing Education, Tirunelveli.


98
 x{A cos ( 2 log x)  ( B sin ( 2 log x)}

1
P.I .  ez
3D  2 D  1
2

1
 e z by § 4(a)
3  2 1

ez x
  .
2 2

y = C.F. + P.I.

 x[ A cos ( 2 log x)  B sin ( 2 log x)  1 / 2]

d2y dy
Ex.2. Solve x 2 2
x  y  log x.
dx dx
Soln:
d
Putting z = log x and D  the equation becomes
dz
[D (D-1) + (D+1)] y = z
i.e., (D2+1) y = z

The auxiliary equation is m2 + 1 = 0

C.F. = A cos z + B sin z

= A cos (log x) + B sin (log x).

1
P.I .  z  (1  D 2 ) 1 z
D 1
2

= (1 – D2 + ...) z = z.

Manonmaniam Sundaranar University, Directorate of Distance & Continuing Education, Tirunelveli.


99
 y = A cos (log x) + B sin (log x) + log x

d2y dy 1
Solve x 2
 3x  y 
dx 2
dx (1  x) 2

Soln:

d
Putting z = log x and D  the equation becomes
dz

1
[ D ( D  1)  3D  1] y 
(1  x) 2

1
i.e., ( D 2  2 D  1) y 
(1  x) 2

The auxiliary equation is (m + 1)2 = 0

 m = - 1 twice

1
C.F .  e  z ( A  B z )  ( A  B log x)
x

1 1 d
P.I .  Changing D to the operator   x
(  1) (1  x)
2 2
dx

1 dx
 x 1  by § 8.2
(  1) (1  x) 2

1 1 1

(  1) x 1  x

dx 1 1 
 x 1   x 1     dx
x(1  x)  x 1 x 
Manonmaniam Sundaranar University, Directorate of Distance & Continuing Education, Tirunelveli.
100
1 x
 log .
x 1 x

y = C.F. + P.I.

d2y dy
Solve x 2 2
 4x  2 y  ex.
dx dx
Soln:
d
Putting z = log x and D  the equation becomes
dz

(D2 + 3D + 2) y = ex

The auxiliary is m2 + 3m + 2 = 0.

 m = -1 or -2.

C.F. = A e-z + B e-z = A x-1 + B x-2.

1 d
P.I .  e x , where   x
(  1) (  2) dx

 1 1  x
  e
  1   2 

x
 x 1  e x dx  x 2  x e dx by § 8.2.

= x-1 ex – x-2 (xex – ex)

= x-2 ex.

y = A x-1 + B x-2 ex + x-2 ex.


Manonmaniam Sundaranar University, Directorate of Distance & Continuing Education, Tirunelveli.
101
d2y dy log x sin (log x)  1
Solve x 2 2
x y
dx dx x
Soln:

d
Putting z = log x and D  the equation becomes
dz
( z sin z  1)
( D  1) 2 y  .
ez
The auxiliary equation is (m - 1)2 = 0; m = 1 twice.

C.F. = e-z (A + B z) = x (A + B log x).

1
P.I .  [ z sin z  1] e  z
( D  1) 2

1
 ez [ z sin z  1] by § 4(d)
( D  2) 2

 1 1
 x 1 Im aginary part of z eiz  
 ( D  2) 2
4

 1 1
 x 1  I .P. of e iz z 
 ( D  i  2) 2
4

 1  2D  1
 x 1  I .P. of eiz 2 
1 z  
 (i  2)  i  2  4

 1  2  1
 x 1  I .P. of  z  (i  2)  
 3  4i  5  4

 1  4  6  1
 x 1   3z   sin z   4 z   cos z   
 25  5  5  4

Manonmaniam Sundaranar University, Directorate of Distance & Continuing Education, Tirunelveli.


102
1 6 4 1
  cos (log x)  sin log x  log x {4 cos (log x)  3 sin (log x)}
100 x 125x 125x 25x

y = C.F.+ P.I.

Manonmaniam Sundaranar University, Directorate of Distance & Continuing Education, Tirunelveli.


103
UNIT – 4 : LAPLACE TRANSFORM
Laplace transform – Inverse transform – Properties-Solving differential equations.
Simultaneous equations of first order using Laplace transform.

THE LAPLACE TRANSFORMS

1.1. Definition: If a function f(t) is defined for all positive values of the variable t and

if  e  st f (t ) dt exists and is equal to F(s), then F(s) is called the Laplace transform of f(t) and
0

is denoted by the symbol L{f(t)}.


Hence L {f(t)} =  e st f (t ) dt  F ( s). The operator L that transforms f(t) into F(s) is
0

called the Laplace transform operator.

Note: Lt F(s) = 0.
S 

Definitions. Piecewise continuity.

A function f (t) is to be piecewise continuous in a closed interval [a,b]if it is defined on that


interval and is such that the interval can be broken up into a finite number of sub-intervals in
each of which f (t) can have only ordinary finite discontinuities in the interval.

Exponential order.

A function f(t) is said to be of exponential order if Lt e-st f(t) = 0, or if for some number s0,
S 

the product ⃒f(t) ⃒<M for t>T,i.e.,e-s0t ⃒f(t) ⃒ is bounded for large value of t,say for t > T.

Sufficient conditions for the existence of the Laplace transform.

(i) f(t) is continuous or piecewise continuous in the closed interval [a,b], where a>0

(ii) it is of exponential order

Manonmaniam Sundaranar University, Directorate of Distance & Continuing Education, Tirunelveli.


104
(iii) tn f(t) is bounded near t = 0 for some number n>1.

From the definition the following results can easily be proved:-

(i) L {f(t) +  (t)} = L{f(t)} + L{(t)}

Proof:

We have L {f(t) +  (t)} =  e st [ f (t )   (t )] dt
0

 
=  e st f (t ) dt   e st (t ) dt
0 0

L = {f(t)} L{ (t)}.

(ii) L{c f(t)} = cL {f(t)}, where c is a constant

Proof:


We have L {c f(t)} =  e st c f (t ) dt
0


= c  e  st f (t ) dt
0

= cL{f(t)}

(iii) L {f' (t)} = sL{f(t)}– f(0).

Proof:

We have L{f' (t)}  e st f ' (t ) dt
0

Manonmaniam Sundaranar University, Directorate of Distance & Continuing Education, Tirunelveli.


105

    f (t ) (s) e
= f(t) e st

0
 st
dt (on integration by parts)
0


= - f(0)  s  f (t ) e st dt
0

= s L{f(t)}- f(0)

(iv) L{f'(t)} = s2L {f(t)} – s f(0) – s f' (0)

Poof:

L {f"(t)} = L {F'(t)}, where F(t)  f' (t)

= s L{F(t)}- F(0)  s L {f' (t)}- f' (0)

= s [s L {f(t)}- f(0)] - f' (0)

= s2L {f(t)}- s f(0) - f' (0).

(v) By extending the previous result, we get

L{ f n (t )} s n L{ f (t )} s n1 f (0)  s n2 f ' (0) ...  f n1 (0)

(vi) If L{f(t)}  F(s), then

(a) lim f (t )  lim s F (s).


t 0 s 

(b) lim f (t )  lim s F (s).


t  s 0

Proof:

L{ f ' (t )} s L{ f (t )} f (0)

Manonmaniam Sundaranar University, Directorate of Distance & Continuing Education, Tirunelveli.


106
 s F (s)  f (0)

Taking limits as s →∞ on both sides, we get

lim [s F (s)  f (0)]  lim L{ f ' (t )}


s  s 


 lim  e st f ' (t ) dt
s 
0

= 0.

lim s F (s)  f (0)


s

 lim f (t )
t0

This result is known as Initial value theorem.

Taking limits as s →0 on both sides of L {f’ (t)}, we get


lim [ s F ( s)  f (0)]  lim  e st f ' (t ) dt
s 0 s 0
0


  f ' (t ) dt
0

 [ f (t )]0

 lim f (t )  f (0).
t 

Manonmaniam Sundaranar University, Directorate of Distance & Continuing Education, Tirunelveli.


107
lim s F (s)  lim f (t )
s 0 t 0

This result is known as final value theorem.

1
(Vii) L (e-at )  provided s  a  0.
sa

Proof:

L (e  at
)   e  st e  at dt
0


  e  ( s  a ) t dt
0


 e  ( sa ) t  1
   .
  ( s  a)  0 s  a

1
(Vii) L (eat )  provided s  a  0.
sa
Proof:

L (e )   e  st e at dt
at


  e  ( s a ) t dt
0


 e ( s  a ) t  1
   .
  ( s  a)  0 s  a

s
Corolary:. L (cosh at) 
s  a2
2

Manonmaniam Sundaranar University, Directorate of Distance & Continuing Education, Tirunelveli.


108
Proof:

We have
 eat  eat 
L (cosh at) = L  
 2 
1 1
 L (e at )  L (e at )
2 2

1 1 1 1
 
2 sa 2 sa

s

s  a2
2

a
Corolary:. L (sinh at) 
s  a2
2

Proof:

We have
 e at  e at 
L (sinh at) = L  
 2 
1 1
 L (e at )  L (e at )
2 2

1 1 1 1
 
2 sa 2 sa

a

s  a2
2

a
L (sinh at) =
s - a2
2

s
(Viii) L (cos at) =
s  a2
2

Manonmaniam Sundaranar University, Directorate of Distance & Continuing Education, Tirunelveli.


109
Method1.

Proof:

We have

L (cos at) =  e -st cos at dt
0


 e -st (- s cos at  a sin at) 
= 
 s2  a2 0
s
=
s  a2
2

Method 2.

L (cos at) = real part of  e -st e ait dt
0

= real part of L (eait)

1
= real part of
s - ai

s  ai
= real part of
s2  a 2
s
=
s  a2
2

Method 3. Let f(t) be cos at.

Then f' (t) = a sin at, f (t) = -a2cos at.

We have L {f (t)} = s2L {f(t)} – s f(0) – f'(0).


 L {- a2cos at} = s2L{f(t)} – s f(0) - f' (0).

We have f(0) = 1, f' (0) = 0.

 - a2L (cosat) = s2L (cos at) – s


Manonmaniam Sundaranar University, Directorate of Distance & Continuing Education, Tirunelveli.
110
i.e., (s2 + a2) L (cos at) = s.
s
L (cos at ) 
s  a2
2

a
(ix) (L sin at) 
s2  a 2
Proof:

We have L (sin at)   e st sin at dt
0


 e  st ( s sin at  a cos at ) 
 
 s2  a2 0

a
=
s  a2
2

Aliter. Let f(t) be sin at, then f' (t) = a cos at.

We have L {f'(t)} = s L{f(t)} – f(0)

i.e., L (a cos at) = s L (sin at) – 0

s
i.e., a  s L (sin at )  0
s  a2
2

a
L (sin at ) 
s  a2
2

(n  1)
(x) L (t n )  .
s n1

Proof: We have L (t )   e st t n dt n

1
Put st = x, then dt  dx.
s
 n
 x 1
L (t )     e  x dx
n

0  
s s
Manonmaniam Sundaranar University, Directorate of Distance & Continuing Education, Tirunelveli.
111

1
s n1 0
 x n e  x dx

(n  1)

s n1
When n is a positive integer Γ(n+1) = n!

n!
L (t n )  when n is a + ve integer
s n1
1
Cor. L(1) 
s

1
L(t ) 
s2

2
L(t 2 ) 
s3

1 1
( )
 (3/2) 
L(t )  3 / 2  3 / 22  3 / 2
1/ 2 2
s s 2s

1
( )

L(t 1 / 2 )  1 /22  1 / 2 .
s 2s

Examples:

Ex.1. Find L (t2 + 2t + 3)

Soln:

L (t2+2t+3) = L(t2) + 2L (t) + 3L (1)

2 2 3
   .
s3 s 2 s

Manonmaniam Sundaranar University, Directorate of Distance & Continuing Education, Tirunelveli.


112
Ex.2. Find L(sin2 2t).

Soln:
 1  cos 4t 
Since , sin 2 2t   , we have
 2 
 1  cos 4t 
L (sin 2 2t )  L  
 2 

1 1
 L (1)  L (cos 4t )
2 2

11 1 s
 
2 s 2 s 2  42

11 s 
   2 
2  s s  16 

8

s ( s  16)
2

Ex.3. Find L(sin3 2t).


Soln:

Since sin 6t = 3 sin 2t – 4 sin3 2t, we have


 3sin 2t  sin 6t 
L (sin 3 2t )  L  
 4 

3 1
 L (sin 2t )  L (sin 6t )
4 4

3 2 1 6
 
4 s  2 4 s 2  62
2 2

Manonmaniam Sundaranar University, Directorate of Distance & Continuing Education, Tirunelveli.


113
48
 .
( s  4) ( s 2  36)
2

Ex.4. Find L {f(t)}, where

F(t) = 0 when 0 <t ≤ 2

= 3 when t > 2.

Soln:

We have

L{ f (t )}  e st f (t ) dt
0

2 
e  st
f (t ) dt   e st f (t ) dt
0 2

2 
  e (0) dt   e st (3) dt
 st

0 2


 3  e  st dt
2

3
=𝑠 𝑒 −2𝑠

1.2 The inverse transforms.


Let the symbol L-1 {F (s)} denote a function, whose Laplace transform is F(s).Thus if
L {f(t)} = F(s) then f(t) = L-1 {F(s)}.

The most obvious way of finding the inverse transform of a given function is to look
into the table of transforms and get the function whose Laplace transform is the given
function.

We can compile the table of transforms from the known results.

Manonmaniam Sundaranar University, Directorate of Distance & Continuing Education, Tirunelveli.


114
s.no f(t) F(s)

1
1. eat
sa
s
2. cosh at
s  a2
2

a
3. sinh at
s  a2
2

s
4. cos at
s  a2
2

a
5. Sin at
s  a2
2

1
6. 1
s
1
7. T
s2
n!
8. tn (n is a  ve int eger )
s n 1
1
9. t eat
( s  a) 2
2
10. t2 eat
( s  a) 3
n!
11. tn eat (n is a  veint eger )
( s  a) n1
b
12. e-at sin bt
( s  a) 2  b 2
sa
13. e-atcosbt
( s  a) 2  b 2
2as
14. t sin at
(s  a 2 ) 2
2

s2  a2
15. t cos at
(s 2  a 2 ) 2
We can modify the results we have obtained in finding the Laplace transforms of
functions to get the inverse transforms of functions.
Manonmaniam Sundaranar University, Directorate of Distance & Continuing Education, Tirunelveli.
115
(i) If L {f(t)} = F(s), then L {e-atf(t)} = F (s+a).
Hence we get the result

L-1 {F(s+a)} = e-at f(t)}

= e-at L-1 F(s).


Thus for example

 1  at 1  1  at
1. L1  2
 e L  2   e t.
 ( s  a)  s 

1 1 e 2t sin 4t
2. L1  e  at 1
L  .
( s  2) 2  16 s 2  42 4

s 3  s  3t
3. L1  e3t L1  2   e cos 2t.
( s  3)  4
2
 s  4

 s  1  s 
4. L1  L  2
 s  2s  5   ( s  1)  2 
2

 ( s  1) 1 
 L1  2
 ( s  1)  2 
2

 s 1  1  1 
 L1  2
L  2
 ( s  1)  2   ( s  1)  2 
2 2

 s  t 1  1 
 e t L1  2 2 
e L  2 2 
s 2  s 2 

sin 2t
 e t cos 2t  e t
2
Manonmaniam Sundaranar University, Directorate of Distance & Continuing Education, Tirunelveli.
116
e t
 (2 cos 2t  sin 2t ).
2

1 s
(ii) If L{ f (t )} F ( s), then L [ f (at )]  F  .
a a

This result can be written in the form

 1  s 
L1  F    f (at ), where f (t )  L1 F ( s)
 a  a 

1
Putting  k , we have
a

1 t
L1 F [(ks)]  f   where f (t )  L1 F ( s).
k k

Examples:

 s 
Find L1  2 2 2
s a  b 
Soln:

s 1 sa 1
 2 2  F ( sa)
s a b a s a b a
2 2
2 2

sa
Where F (sa) =
s a 2  b2
2

s
F (s) 
s  b2
2

Manonmaniam Sundaranar University, Directorate of Distance & Continuing Education, Tirunelveli.


117
 s  1 1  sa 
L1  2 2 2
 L  2 2
s a b  a  s a  b 
2

1 1
 L [ F (as)]
a

1 1 t
 . f  ,
a a a

Where f(t)  L-1 F (s)

s
 L1 .
(s  b 2 )
2

= cosbt.

t  bt 
 f    cos  .
a a
 s  1  bt 
Hence L1   2 2 2
 2 cos  .
s a b  a a
(iii) If L {f(t)} = F (s), then L {tf(t)}= -F' (s)

Hence we get the result

L-1 {F'(s)} = - tf(t) = tL-1 {F(s)}.

Examples.
 s 
Ex.1. Find L-1  2 2 2
s  a ) 

Soln:
 s 
F ' (s)   2 2 2
 (s  a ) 

Manonmaniam Sundaranar University, Directorate of Distance & Continuing Education, Tirunelveli.


118
s ds
F ( s)  
(s  a 2 ) 2
2

1

2( s  a 2 )
2

 s   1 
L1  2 2 2
  tL1  2
 (s  a )   2 (s  a 
2

t 1  1 
 L  2 2 
2 s a 

t
 sin at
2a

 s 
Ex.2. Find L-1  2 2
 (s  1) 

Soln:
s s
Here F ' ( s)  F (s)   2 ds.
( s  1)
2 2
( s  1) 2

1
 .
2( s  1)
2

 s  1
L1  2 2
  tL1 [ ].
 ( s  1)  2( s  1)
2

t  1 
 L1  2 
2  s 1

t
 sinh t.
2

Manonmaniam Sundaranar University, Directorate of Distance & Continuing Education, Tirunelveli.


119
 s2 
Ex.3. Find L-1  2 5
 (s  4s  5) 

Soln:

s2
Here F ' ( s) 
( s  4s  5) 2
2

1
F ( s) 
2 ( s  4s  5)
2

 s2   1 
L1  2 2
  tL1  
 ( s  4s  5)    2 ( s  4s  5) 
2

t 1  1 
 L  2
2  s  4s  5 

t  1 
 L1  2
 ( s  2)  1 
2
2

t  1 
 e 2t L1  2 2 
2  s 1 

t e 2t sin t
 .
2

(iv) If L {f(t)} = F (s), then L {tf(t)} = - F' (s).

This theorem can be used in the following way to get inverse transforms of certain functions:
-
 s  1
example L-1 log .
 s - 1 

Manonmaniam Sundaranar University, Directorate of Distance & Continuing Education, Tirunelveli.


120
Soln:
Let this be equal to f(t).

s 1
Then L {f(t)}  log .
s-2

d s 1
L{tf (t )}  log
ds s 1

d
 [log ( s  1)  log ( s  1)]
ds
1 1
  .
s 1 s 1

 1  1  1 
 tf (t )  L1   L  
 s 1   s 1

= et – e-t

= 2 sinh t.

2 sinh t
 f (t )  .
t

t
1
(v) L [  f ( x) dx]  L [ f (t )].
0
s

Soln:

t
Let  f ( x) dx be F (t )
0

Then F' (t) = f(t) and F (0) = 0


Manonmaniam Sundaranar University, Directorate of Distance & Continuing Education, Tirunelveli.
121
L{F ' (t )}  sL{F (t )}  F (0)
 sL{F (t )}
t
i.e., L{ f (t )} sL{ f ( x) dx}
0

t
1
Hence L  f ( x) dx  L { f (t )}.
0
s

This result can also be used to find the inverse transforms of certain funcitons.

1 
t

 f ( x) dx  L
1
 s { f (t )}
0

If L {f(t)} = F(s), then

1 
t
L1  F ( s)   f ( x) dx,
s  0

Where f(t) = L-1 F(s).

1 
t
L1  F ( s)   L1 {F ( s)}dt.
s  0

Examples.
 1 
Ex.1. Find L-1  
 s (s  a) 

Soln:

 1  t  1 
L1      dt
 s ( s  a)  0  s  a 

t
  e at dt
0

Manonmaniam Sundaranar University, Directorate of Distance & Continuing Education, Tirunelveli.


122
t
 e  at 
 
  a 0

1
 (1  e at ).
a

 1 
Ex.2. Find L-1  2 2 
 s (s  a ) 
Soln:
 1  t 1  1 
L1  2 2 
 L  2 2 
dt
 s (s  a )  0  s  a 

t
sin at
 dt
0
a

1   cos at 
t

 
a  a  0

1 cos at

a2

 1 
Ex.3. Find L-1  2 2 2
 (s  a ) 

Soln:

 1  1 s 
L1  2 2 2
 L1  . 2 2 2
 (s  a )   s (s  a ) 

t
 s 
  L1  2 2 2
dt
0  (s  a ) 

Manonmaniam Sundaranar University, Directorate of Distance & Continuing Education, Tirunelveli.


123
t
t sin at
 dt
0
2a

1   t cos at sin at 
t

   2 
2a  a a 0

1
 (sin at  at cos at ).
2a 3
(vii)The method of partial fractions can be used to find the inverse transform of certain
functions.
The method is illustrated in the following examples.

Examples.

 1 
Ex.1. Find L-1  
 s (s  1)(s  2) 

Soln:
1
We can split into partial fractions as
s (s  1) (s  2)

11 1 1 1
  .
2 s s 1 2 s  2

 1  1 1  1  1  1 
L1   L   L  
 s ( s  1) ( s  2)  2  s   s 1

1  1  1 t 1  2 t
 L1   e  e
2  s2 2 2

 1 
Ex.2. Find L-1  .
 (s  1) (s  2s  2) 
2

Manonmaniam Sundaranar University, Directorate of Distance & Continuing Education, Tirunelveli.


124
Soln:
Splitting into partial fractions, we have

1 1 s 1
  2
(s  1) (s  2s  2) s  1 s  2s  2
2

 1 
 L1  
 ( s  1) ( s  2s  2) 
2

 1  1  s  1 
 L1   L  2 
 s 1  s  2s  2 

 s 1 
 e t  L1  
 ( s  1)  1
2

 s 
 e t  e t L1  2 
 s 1

= e-t – e-tcos t

= e-t (1- cos t)

 1  2s 
Ex.3. Find L-1  .
 (s  2)  ( s  1)
2 2

Soln:

1  2s 1 ( s  2) 2  ( s  1) 2
 .
(s  2) 2 ( s  1) 2 3 ( s  2) 2 ( s  1) 2

1 1 1 
  
3  ( s  1) ( s  2) 2 
2
.

Manonmaniam Sundaranar University, Directorate of Distance & Continuing Education, Tirunelveli.


125
 1  2s 
Hence L-1  2
 (s  2) ( s  1) 
2

1  1  1 1  1 
 L1   L  .
3  ( s  1)2  3  ( s  2)2 

1 1
 (e t t )  (e  2 t t )
3 3

t
 (e t  e 2t ) .
3
Exercises

Find the inverse transforms of :

𝟏 𝒔
1.(𝒔−𝟑)𝟓 2 . 𝟐 .
(𝒔−𝒃)𝟐+𝒂

𝒄𝒔+𝒅 𝒔
3. 𝟐 4.(𝒔+𝟑)𝟓
(𝒔+𝒂)𝟐+𝒃

Laplace transformation can be used to solve ordinary differential equations with


constant coefficients.

Examples.
d2 y dy dy
Ex.1. Solve the equation 2
 2 - 3y = sin t given that y   0 when t  0
dt dt dt
Soln:The equation can be written in the form

y + 2y' – 3y = sin t.

Applying Laplace transforms to both sides, we have

L (y + 2y' – 3y) = L (sin t)


Manonmaniam Sundaranar University, Directorate of Distance & Continuing Education, Tirunelveli.
126
1
i.e., L (y")  2L (y') - 3L (y) 
s 1
2

1
S2 L ( y)  sy (0) y' (0)  2{sL ( y)  y(0)} 3L ( y) 
s 1
2

Substituting the values of y(0) and y' (0) in the equations

1
s 2 y  2s y - 3y  , where y  L ( y).
s 12

1
(s 2  2s - 3) y 
s 1
2

1
y
(s  2s  3) ( s 2  1)
2

1

(s  3) (s - 1) (s 2  1)

1
y  L-1 
(s - 1) (s  3) (s 2  1)

On splitting into partial fractions, we get

 1 1 1 1
  s 
y  L-1  40  8  102 5
 s  3 s 1 s 1 
 
 

1 1  1  1 1  1 
- L   L  
40  s  3  8  s  1 

1 1  s  1 1  1 
- L   L  
10  s 2  1  5  s 2  1 

Manonmaniam Sundaranar University, Directorate of Distance & Continuing Education, Tirunelveli.


127
1 3t 1 t 1 1
- e  e  cos t  sin t.
40 8 10 5
d2 y
Ex.2. Show the solution of the differential equation  4 y  A sin kt which is such that y =
dt 2
k
dy sin kt  sin 2t
0 and  0 when t = 0 is y  A 2 if k  2. If k=2,
dt 4k2
A (sin 2t - 2t cos 2t)
show that y 
8
Soln:
y + 4y = A sin kt

L (y) + 4L (y) = A L (sin kt)

k
s 2 y  sy(0)  y' (0)  4 y  A , where L ( y)  y .
s  k2
2

Since y (0) = 0, y' (0) = 0, we have

k
(s 2  4) y  A. .
s  k2
2

k
y  A.
(s  4) ( s 2  k 2 )
2

1
y  A K L-1
(s  4) ( s 2  k 2 )
2

Case( i). If k ≠ 2,
 1 1 
 2  2 
y  A K L-1  s  4 2 s  k
2

 (k  4) 
 
 

Manonmaniam Sundaranar University, Directorate of Distance & Continuing Education, Tirunelveli.


128
A K  1 1 1 
 L 2  L1 . 2 
k 4 s 4
2
s  k2 

A k  sin 2t sin kt 
   
k2  4  2 k 

A  k 
 2 
sin kt  sin 2t 
4-k  2 

Case (ii). K = 2. Then


 1 
y  2 AL-1  2 
 ( s  4) ( s  4) 
2

 1 
 2 AL-1  2 2 2
 (s  2 ) 
1
 2A. (sin 2t  2t cos 2t )
2(2)3
A
 (sin 2t  2t cos 2t ).
8
Note:-The special advantage of this method in solving differential equations is that the initial

conditions are satisfied automatically.It is unnecessary to find the general solution and

determine the constsnts using the initial conditions.

1.3 The Laplace transform can also be used to solve systems of differential

equations.

Ex.1. Solve the simultaneous equations.


dx dy
3   2 x 1
dt dt ... (1)
dx dy
 4 3y 0
dt dt …(2)
given x = 0 = y at t = 0.

Manonmaniam Sundaranar University, Directorate of Distance & Continuing Education, Tirunelveli.


129
Soln:
Applying Laplace transforms to both the equations, (1) becomes

3L (x') + L (y') + 2 L (x) = L (1)

1
i.e., 3{s x - x (0)} s y  y (0)  2 x 
s

where x  L (x) , y  L (y).

Since x(0) = 0, y(0) = 0, we have


1
3s x  s y  2 x  .
s
1
i.e., (3s  2) x  s y 
s
Equation (2) becomes

L (x') + 4 L (y') + 3L (y) = 0

i.e., s x  x (0)  4{s y  y (0)} 3 y  0

i.e., s x  (4s  3) y  0

Solving (3) and (4), we get


4s  3
x
s (s  1) (11s  6)
1
y-
(11s  6) ( s  1)

 4s  3 
 x  L1   
 s (s  1) (11s  6) 

Manonmaniam Sundaranar University, Directorate of Distance & Continuing Education, Tirunelveli.


130
1 1 33 
 
 L-1  2  5  10 
 s s  1 11s  6) 
 
 

1  1  1  1  33 1  1 
 L-1    L1   L  
2  s  5  s  1  10  11s  6) 

1 1 3  6t
  e t  e 11
2 5 10

 1 
y  L-1  
 (11s  6) ( s  1) 

 1 1 11 1 
 L-1  .  . 
 5 s  1 5 11s  6 
 
1 1  1  11 1 1  1 
 L   . L  
5  s  1  5 11  s  6 
 
 11 
6t
1 1 
 et  e 11 .
5 5
Ex.2. Solve the simultaneous equations.
dx dy
  2 x  2 y 1 2t
dt dt ... (1)
d 2x dy
2
 2  x 0
dt dt …(2)
dx
With the conditions x = 0, y = 0,  0 when t = 0
dt
Soln:
Applying Laplace transforms to both the equations, equation(1) becomes
L (x' ) - L (y') - 2L (x)  2L (y) L (1) - L (2t)

Manonmaniam Sundaranar University, Directorate of Distance & Continuing Education, Tirunelveli.


131
1 2
i.e., s x  x (0)  s y  y (0)  2 x  2 y   2
s s
1 2
i.e., s x  s y  2 x  2 y   2
s s
s2
i.e., ( s  2) x  ( s  2) y 
s2
1
x-y
s2 ... (3)
Equation (2) becomes

L (x) + 2L (y')+ L (x) = 0

i.e., s 2 x  sx (0)  x' (0)  2s y  2 y (0)  x  0

i.e., s 2 x  2s y  x  0

i.e., (s 2  1) x  2s y  0 ... (4)


Solving equations (3) and (4) for x and y, we get

2 s2 1
x , y  
s( s  1) 2 s 2 ( s  1) 2

 2 
x  L1  2
 s ( s  1) 
1 1 1 
 2 L1    2
 s s  1 ( s  1) 

 1  1  1  1  
 2 L1    L1   L  2 
 s  s 1  ( s  1)  

= 2 (1 – e-t – t e-t).

Manonmaniam Sundaranar University, Directorate of Distance & Continuing Education, Tirunelveli.


132
 s2 1 
y   L1 .  2 2
 s ( s  1) 

t
 s2 1 
   L1  2
dt
0  s ( s  1) 

t
1 2 
   L1 .   2
dt
0  s ( s  1) 
t
   (1  2t e t ) dt
0

= 2 – t – 2 (t+1) e-t.

Laplace transform can be used to solve differential equations with variable coefficients.

We have shown that


d
L{t f (t )}  L{ f (t )}
ds
d2
and {t 2 f (t )} (1) 2 L{ f (t )}.
ds 2

These results are used to solve equations containing variable coefficients.

The following worked out examples will illustrate the method.

Examples.
Ex.1. Solve the equation
d2y dy
t 2
 (2  t )  3 y  t  1when y(0)  0.
dt dt
Soln:
Taking Laplace transforms on both sides, we have

Manonmaniam Sundaranar University, Directorate of Distance & Continuing Education, Tirunelveli.


133
L (ty) – L {(2+t) y'} + 3L (y) = L (t-1)

d 2
i.e.,  {s L ( y)  sy (0)  y' (0)}
ds
d 1 1
 2{sL ( y)  y(0)} {sL ( y)  y(0)} 3L ( y)  2  .
ds s s
Let L ( y) be y .

Putting y(0) = 0,

d 2 d 1 s
 {s y  y' (0)} 2 ( s y )  ( s y )  3 y  2
ds ds s

dy dy 1 s
i.e.,  s 2  2 s y  2s y  s  y 3 y  2
ds ds s

dy s 1
i.e.,  ( s 2  s)  4 ( s  1) y   2
ds s

d y 4y 1
i.e.,  
ds s s3

1 1 c
Solving this equation, y  . 2  4 .
2 s s

1 1 1
 y  L1  2   c L1  4 
s s  s 
t c t3
  .
2 6
t
Hence y   A t 3 , Where A is an arbitrary constant.
2

d 2 y dy
Ex.2. Solve the equation. dt 2  t dt  y  0 if y (0)  0 and y ' (0) 1.

Manonmaniam Sundaranar University, Directorate of Distance & Continuing Education, Tirunelveli.


134
Soln:
Taking Laplace transforms on both sides, we have

L (y) + L (t y') – L (y) = 0

d
i.e., s 2 L ( y)  sy (0)  y' (0)  {sL ( y)  y (0)} L ( y)  0
ds

y(0) = 0, y' (0) = 1.

d
Putting L ( y)  y , we get s 2 y  1 (s y )  y  0
ds
dy
i.e., s 2 y  1  s  y  y 0
ds
dy
i.e., s  ( s 2  2) y 1 0
ds

Solving this equation y s 2 e  s  es  c.


2 2
/2 /2

es / 2
2
1
y  2  c 2 .
s s
y is a Laplace transform.

lim y  0,c  0.
s 

1
Hence y 
s2
Taking inverse transform y = t.

Certain equations involving integrals can also be solved by Laplace transform


Example. Determine y which satisfies the equation
t
dy
 3 y  2 y dt  t for which y (0)  0.
dt 0

Manonmaniam Sundaranar University, Directorate of Distance & Continuing Education, Tirunelveli.


135
Soln:
Taking Laplace transforms on both sides, we get
t
L ( y ' )  3L ( y )  2 L (  y dt )  L (t )
0

2 1
i.e., s L ( y)  y (0)  3L ( y)  L ( y)  2
s s

Putting L (y) = y and substituting y(0) = 0, we have

2 1
s y 3 y  y  2
s s

 2  1
i.e., y  s  3   2
 s  s

 2 1
i.e., y  s  3    2
 s s

1
y 
s ( s  1) ( s  2)

 1 
 y  L1  
 s ( s  1) ( s  2) 

1 1 1 1 1 
y  L1  .   . 
2 s s 1 2 s  2 

1 1  1  1 1  1 
 L1    L1   L  
2 s  s 1 2  s  2 

1 1
  e t  e 2t .
2 2

Manonmaniam Sundaranar University, Directorate of Distance & Continuing Education, Tirunelveli.


136
UNIT – 5 : PARTIAL DIFFERNTIAL EQUATIONS
Partial differential equations of first order – formation – different kinds of solution – four
standard forms- Lagranges method.

1.1 PARTIAL DIFFERNTIAL EQUATIONS

Partial differential equations are those which involve one or more parital derivatives.

The order of a partial differnential equation is determined by the highest order of the partial

derivativeoccuring in it. For the present, we shall restrict ourselves to partial differential

equations involving one dependent variable z and only two indepandent variables x and y. In

what follows, we shall donote

z z 2z 2z 2z


= p, = q, 2 = r, = s and 2 = t.
x y x x y y
Derivation of partial differential equations.
Parrial differential equations can be derived either by the elimination of (1) arbitrary
constants from a relation between x, y, z
Or (2) of arbitrary function of these variables.

By elimination of arbitrary constants.

Consider the function


f ( x, y, z, a, b)  0 .............. (1)

Containing two independent arbitrary constants a and b. To eliminate two constants,

we require three equations. Differentiating equation (1) partially with respect to x and y in

turn, we obtain
f f
 p 0 ............... (2)
x z
f f
 p 0 ............... (3)
y z

Eliminating a and b, we get a partial differential equation of the first order of the form

F( x, y,z,p,q) = 0.

Examples.

Ex. 1. Eliminate a and b form z = (x+a) (y+b).

Manonmaniam Sundaranar University, Directorate of Distance & Continuing Education, Tirunelveli.


137
Soln:

Differentiating partially with repect to x


dz
 y b
dx
Ie) p=y+b
Again ,Differentiating partially with repect to y
dz
 xa
dy

p = y+b, q =𝑑𝑦x+a.
𝑑𝑥
Eliminating Differentiating partially with repect to x a and b , we get z = pq

Ex.2. Obtain the partial differential equation of all spheres whose centres lie on the plane z

=0 and whose radius is constant and equal to r.

Soln:

The Cartesian equation of all such spheres can be written in the form

(X- a)2 + (y-b)2 + z2 = r2 …………..(1)

Where a and b are independent arbitrary constants and r is the fixed given constant.

Differentiating (1) partially with respect to x and y in turn,we obtain

2(x-a)+2z (dz/dx)=0

(x-a) + pz =0 …………(2)

(y-b) + qz =0 …………(3)

Eliminating a and b between equations (1), (2) and (3), we obtain

Z2 (p2 + q2 + 1) = r2

By the elimination of arbitrary functions.

Let u and v be any two functions of x,y,z and connected by arbitrary relation

(u,v) = 0 ………….(1)

Differentiating equation (1) partially with respect to x and y in turn,we get

Manonmaniam Sundaranar University, Directorate of Distance & Continuing Education, Tirunelveli.


138
  u u    v v 
  p   p  0
u  x z  v  x z 
  ………..(2)
  u u    v v 
  q     q   0
u  y z  v  y z  ………..(3)
 u u 
  between equations (2) and (3),we get a partial
Eliminating the ratio  y z 
differential equation of the first order, viz.,

Pp + Qq = R ………..(4)

Where
u v u v  u, v 
P   .
y z z y   y, z 

u v u v  u , v 
Q   .
z x x z   z , x 

u v u v  u, v 
R   .
x y y x  x, y 

Equation (4) is known as Lagrange’s linear equation.

Examples:

Ex. 1.Elimiate the arbitrary function from z = f(x2+y2)

Soln:

z = f(x2+y2) ............... (1)

Differentiating partially with respect to x and y

p = f′(x2+y2)2x ............... (2)

q = f′ (x2+y2)2y ............... (3)

Eliminating f′ (x2+y2) from (2) and (3), we get py = qx.

Ex. 2 Eliminate the arbitrary functions f and  from the relation Z = f(x+ay) +  (x-ay).

Soln:

Given Z = f(x+ay) +  (x-ay). …………………..(1)


Manonmaniam Sundaranar University, Directorate of Distance & Continuing Education, Tirunelveli.
139
Differentiating partially with respect to x and y

p = f′(x+ay)+′(x-ay) ............... (2)

q = af′ (x+ay) -a′ (x-ay) ............... (3)

Differentiating these again, with respect to x an y


𝜕2𝑧
= 𝑓 ′′ 𝑥 + 𝑎𝑦 + 𝜙 ′′ 𝑥 − 𝑎𝑦 ................ (4)
𝜕𝑥 2
𝜕2𝑧
= 𝑎2 𝑓 ′′ 𝑥 + 𝑎𝑦 + 𝑎2 𝜙 ′′ (𝑥 − 𝑎𝑦)  ................ (5)
𝜕𝑥 2

2 z 2  z
2
From (4) and (5), we get  a
y 2 x 2
Excersises
1. Obtain a partial differential equation by eliminating a,b from each of the following :-
i) Z = ax +by +a
x2  y2 z 2
ii)  2 1
a2 b

Differential integrals of partial differential equations.

A solution or integral of a partial differential equation is a relation between the dependent


and the independent variables that satisfies the differential equation. It will be noted that two
types of solutions may occur as solutions of the same equation. For example, consider the
equations
Z = ax + by …..(1)
𝑦
and z = xf(𝑥 ) …..(2)

If we eliminate the arbitrary constants a and b from the equation (1) and the arbitrary
function from the equation (2),we get the same differential equation xp + yq =z

𝑦
Hence z = ax + by and z= xf(𝑥 ) are solutions of the equation

Xp + yq= z.
A solution containing as many arbitrary constants as there are independent variables
is called a complete integral.
A solution obtained by giving particular values to the arbitrary constants in a
complete integral is called particular integral.

Manonmaniam Sundaranar University, Directorate of Distance & Continuing Education, Tirunelveli.


140
Singular Integral.
Let F (x, y, z, p, q ) = 0 ……(1)

be the partial equation whose complete integral is


(x, y, z, a,b)= 0 …..(2)

The eliminant of a, between


 (x, y, z,a, b) = 0

0
a

0
b
When it exists ,is called the singular integral.
Geometrically, this represents the envelope of the two parameter surfaces represented
by the complete integral (2) of (1).The two parameters occurring in (2) are a and b.

General Integral
In (2), we shall assume an arbitrary relation of the form b= f(a). Then (2) becomes

f[x, y, z, a f(a) = 0 …….(3)


Differentiating (2), partial with respect to a,

 
 f ' (a)  0 ………(4)
a b
The eliminant of a between these two equations (3) and(4), if it exists, is called the general
integral of (1).
Solution of partial differential equation in some simple cases.
We shall consider a number of simple exaples, the solutions of which depend only on
the meaning of the partial differentiation.
Examples.
2z
Ex. 1 Slove  0.
y y

Manonmaniam Sundaranar University, Directorate of Distance & Continuing Education, Tirunelveli.


141
Soln:
2 z   z 
  
y y x  y 

z
Hence  f ( y ) whenf is an arbitrary function.
y

 z   f ( y) dy  φ( x)

= F (y) + (x).
Here F(y) and (x) are arbitrary functions.

2z
Ex. 2 Solve  sin y .
y 2
Soln:
2z
 sin y
y 2
Integrating on,
z
  cos y  f ( x) .
y
Again integrating,
.˙. Z = -siny + yf(x) + (x), where f and  are arbitrary functions.
z
Ex.3 Slove x  y  0.
x
z
Soln: x  y 0
x

z x

x y

Integrating on,

𝑥2
. ˙. 𝑧 = − +𝜙 𝑦 .
2𝑦

2 z
Ex.4 Solve  x2  y2
x y

Soln:

Manonmaniam Sundaranar University, Directorate of Distance & Continuing Education, Tirunelveli.


142
2 z
 x2  y2
x y

Integrating on,
𝜕𝑧 𝑦3
= 𝑥2 𝑦 + +𝜙 𝑥 .
𝜕𝑥 3

Again Integrating on,


x 3 y xy 2
z   F ( x)  f ( y )
3 3
z
Ex. 5 Solve x  2x  y  3z
x
Soln:
z
x  2x  y  3z
x
Dividing by x,

z
 2  y/x  3z/x
x

The equation can be written in the forms


z z y
3  2
x x x

This is a linear equation.

The integration factor is 1/x3


1  z z 2 y
Hence 3 
3   3  4
x  x x x x

  z  2 y
i.e.,  3 3  4 .
x  x  x x
𝑧 1 𝑦
. ˙. = − − + 𝜙(𝑦)
𝑥3 𝑥 2 3𝑥 3

y
i.e., z   x   x3 f ( y)
3

Manonmaniam Sundaranar University, Directorate of Distance & Continuing Education, Tirunelveli.


143
2 z
 a2 z
Ex.6 Solve x 2
given that when x = 0.
z z
 a sin y and 0
x y

If Z is a function of x alone, the solution would be


Z  A eax  B e ax ,where A and B constants.
Soln:
Here z is a function x and y; hence the solution of the equation is

𝑧 = 𝑓 𝑦 𝑒 𝑎𝑥 + 𝜙 𝑦 𝑒 −𝑎𝑥

𝜕𝑧
= 𝑓 𝑦 𝑎 𝑒 𝑎𝑥 − 𝜙 𝑦 𝑎 𝑒 −𝑎𝑥
𝜕𝑥

𝜕𝑧
= 𝑓 ′ 𝑦 𝑒 𝑎𝑥 + 𝜙 ′ (𝑦)𝑒 −𝑎𝑥
𝜕𝑦

z
When x = 0,  a sin y
x
.˙. a f(y) - a  (y) = a siny
i.e., f(y) - (y) = sin y .......... (1)
z
When x = 0 , 0
y
.˙.f′(y) + ′(y) = 0 ........... (2)
Differentiating (1), we get
f′(y) - ′ (y) = cos y ............ (3)
1 1
From (2) and (3), 𝑓 ′ 𝑦 = 𝑐𝑜𝑠𝑦, 𝜙 ′ 𝑦 = − 2 𝑐𝑜𝑠𝑦.
2
1 1
∴ 𝑓 𝑦 = 2 𝑠𝑖𝑛𝑦 + 𝐴, 𝜙 ′ 𝑦 = − 2 𝑠𝑖𝑛𝑦 + 𝐵.

But from (1), A = B.


1 1
Hence z  sin y eax  sin y e ax  A eax  A e ax
2 2
= sinysinhax + 2A coshax.

Manonmaniam Sundaranar University, Directorate of Distance & Continuing Education, Tirunelveli.


144
1.2 Standard types of first order equations
Many of the important equations of the order that occur in practice are one or the
other of the following standard forms.
1  z z 2 y
Hence 3 
3   3  4
x  x x x x
Standard 1. The variable x,y,z do not occur explicitly. Such equations are of the form
f(p,q) = 0 where
z z
p ,q
x y

We can easily verify that z = ax+ by + c is a solution of the equationf(p,q)= 0


provided f(a,b) = 0.
Solving this for b, b = F (a)
Hence the complete integral is z = ax + y F(a) +c.
This singular integral is obtained by eliminating a and c between
z = ax + y F(a) + c
0 = x + y F′ (a)
0 = 1.
The last equation is absurd and shows that there is no singular integral in this case.
To obtain the general integral, we assume an orbitrary relation c =  (a). Then z = ax

+ y F (a) +  (a).

Differentiating partially with respect to a,


0 = x + y F′ (a) + ′ (a).

The eliminant of a between these equations is the general integral.


Note :- The singular and general integral must be indicated besides the complete integral in
every equation. Then only it is said to be completely solved.

Example. Solve p2+q2 = npq.


The solution is z = ax + by + c, where a2+b2 = nab.

Solving b 
a(n  n 2
4 
2
The complete integral is
Manonmaniam Sundaranar University, Directorate of Distance & Continuing Education, Tirunelveli.
145
ay
y  ax  (n  (n 2  4)  c
2

Differentiating partially with respect to c, we see that there is no singular integral, as


we get an absurd result.
To find the general integral put c = f (a), where f is arbitrary.
ay
z  ax  (n  (n 2  4) )  f (a) .
2

Differentiating partially with respect to a,


y
0 x (n  (n 2  4)  f ' (a)
2

The eliminant of a between these equations gives the general integral.


Excercies
Solve the following equations :-
1. p2+q2 = 4. 2. p = q2..
3. pq = 1. 4. pq+p+q =0.
5. q2-3q+p = 2. 6. 3p2-2q2 = 4pq
7. p3-q3 = 0. 8. p + q = 3/a
Standard form 2. Only one of the variables x,y,z occurs explicity. Such equations can be
written in one the forms
F(x, p, q) = 0, F(y, p, q) = 0, F(z, p, q)= 0
(i) Let us consider the form F (x, p, q) = 0
Since z is a function x and y.
z z
dz  dx  dy .
x y

= p dx + qdy.
Let us assume that q = a
The equation becomes F(x, p, a) = 0
Sloving this for p, we get p =  (x, a)

.˙.dz = (x,a)dx + a dy.


𝑧= 𝜙 𝑥, 𝑎 𝑑𝑥 + 𝑎𝑦 + 𝑏.
This consists of two arbitrary constants a and b and hence it is a complete integral.

Manonmaniam Sundaranar University, Directorate of Distance & Continuing Education, Tirunelveli.


146
(ii) Let us consider the form F (y, p,q) = 0
Let us assume that p = a
.˙.F(y, a, q) = 0.
.˙.q = (y,a)
Hence dz = adx + (y,a) dy
.˙. 𝑧 = 𝑎𝑥 + 𝜙 𝑦 𝑎 𝑑𝑦 + 𝑏, Which is complete integral.
(iii) Let us consider the equation F(z,p,q) = 0
Let us assume that q = ap.
Then the equation become F (z,p,ap) = 0
i.e, p =  (z,a)
Hence dz =  (z,a) dx + a  (z,a) dy

𝑑𝑧
i.e, 𝜙 (𝑧,𝑎) = 𝑥 + 𝑎𝑦 + 𝑎𝑑𝑦
𝑑𝑧
i.e., ʃ 𝜙(𝑧,𝑎) = 𝑥 + 𝑎𝑦 + 𝑏 which is a complete integral.

Examples.
Solve
(i) q = xp + p2
(ii) p = y2q2
(iii) p(1+q2) = q(z-1)
(i) q = xp + p2

Soln:
Let q = a
Then a = xp + p2
i.e. p2 + xp - a = 0.

 x  ( x 2  4a )
p
2

 x  ( x 2  4a )
Hence dz  dx  ay  b
2

 x  ( x 2  4a )
  dx  ay  b
2
Manonmaniam Sundaranar University, Directorate of Distance & Continuing Education, Tirunelveli.
147
 x2  x  x 
   (4a  x 2 )  a sinh 1    ay  b.
4 4  2 a 

(ii) p = y2q2

Soln:
Let p = a2
.˙. q =  a/y

Hence dz = a2 dx  (a/y) dy

.˙. z = a2x  a log y + b.

(iii) p (1+q2) = q (z-1).

Soln:
Let q = ap.
Then p(1 + a2p2) = ap (z-1)
ie., 1+a2p2 = a (z-1)
(az  a  1)
p  .
a
(az  a  1)
Hence dz   dx  az  a  1 dy
a
a dz
i.e.,   dx  a dy
(az  a  1)

a dz
i.e.,    xa y b
(az  a  1)

i.e.,  2 (az  a  1)  x  a y  b.

Excersies
Solve the following equations :-
1.p = 2qx 2. q = 2yp2
2 2
3. 9(p z +p ) = 4. 4.p (1+q) = qz

Manonmaniam Sundaranar University, Directorate of Distance & Continuing Education, Tirunelveli.


148
Standard form 3
Equations of the form f1(x,p) = f2 (y, q).

In this form the equations is of the first order and the variables are separable. In the
equations z does not appear. We shall assume as a tentative solution that each of these
quantities is equal to a.
f1(x,p) = a, Solving p = 1 (a, x).
f2(y,q) = a, Solving q = 2 (a, y).
Hence dz = 1 (a,x)dx + 2(a,y) dy.
∴𝑧= 𝜙1 𝑎, 𝑥 𝑑𝑥 + 𝜙2 𝑎, 𝑦 𝑑𝑦 + 𝑏which is a complete integral.

Example. Solving the equation p +q = x + y.


Soln:
We can write the equation in the form p-x = y-q.
Let p-x = a. Then y-q = a.
Hence p = x+a. q = y-a
.˙. dz = (x+a) dx + (y-a) dy.

(𝑥+𝑎)2 (𝑦 −𝑎)2
.˙. 𝑧 = + + 𝑏.
2 2

There is no singular integral and the general integral is found as usual.

Standard 4. Clairant’s form

This is of the form z = px + qy + f(p,q).


The solution of the equation is z = ax + by + f(a,b) for p = a and q = b can easily be
verified to satisfy the given equations.

Example. Solve z  px qy  (1  p 2  q 2 )

The complete integral is obviously

z  ax  by (1  a 2  b 2 ) .

To find the singular integral, differentiating partially with respect to a and b,


Manonmaniam Sundaranar University, Directorate of Distance & Continuing Education, Tirunelveli.
149
a b
x  0 and y  0.
1  a2  b2 1  a2  b2
Eliminating a and b the singular integral is x2+y2+z2 = 1.
To find the general integral, assume b = f (a), where f is arbitrary.
Then z = ax+ f(a) y + {1+ a2 + (f (a)2)}1/2
Differntiating partially with respect to a and eliminate a between the two eqations.

Excersies 26
Solve the following equations:-
1. z = px +qx + pq
2. z  px  qy  2 pq

p
3. z  px  qy   p.
q
1.3 LAGRANGE’S EQUATION.

We have shown that, if we eliminate the arbitrary function F from the relation F(u,v)
= 0, where u and v are functions of x,y,z. we get the equations
(u, v) (u, v) (u, v)
p q .
 ( y, z )  ( z , x) ( x, y)
This is expressed in the form Pp + Qq = R, where P,Q and R are function of x,y,z.
This partial differential equation is known as Lagrange’s equation.
In the following article we shall try to find the solution of the equation Pp + Qq = R.

The general solution of the partial differential equation Pp + Qq = R is F (u,v) = 0


where F is an arbitrary function and u (x,y,z) = C1 and v (x,y,z) = C2 from two independent
dx dy dz
solutions of the equations   .
P Q R
Taking total differential on the equation u (x,y,z) = C1 we get
u v u
dx  dy  dz  0 .
x y z
Since u(x,y,z) = C1 is a solution of the equation
dx dy dz
 
P Q R

Manonmaniam Sundaranar University, Directorate of Distance & Continuing Education, Tirunelveli.


150
u u u
P Q  R  0. ............ (1)
x y z
Similarly, v(x,y,z) = C2 is a solution of the equation
dx dy dz
 
P Q R
v v v
P  Q  R  0. ............ (2)
x y z
From equation (1) and (2), we get
P Q R
  .............(3)
u v u v u v u v u v u v
  
y z z y z x x z x y y x

P Q R
i.e.,  
 (u, v)  (u, v)  (u, v)
 ( y , z )  ( z , x )  ( x, y )

We have shown that the elimination of the arbitrary function F from the equation
F(u,v) = 0, where u and v are functions of x,y,z leads to the partial differential equation

(u, v) (u, v) (u, v)


p q  .......... (4)
 ( y, z ) ( z, x) ( x, y )
Substituting from equation (3) in (4), we get the equation
pP + qQ = R ........... (5)
Hence we see that F(u,v) = 0 is a solution of the equation (5).
dx dy dz
If u = c1 and v = c2 are the solutions of  
P Q R
Examples.

Ex. 1 Solve (y2 + z2)p - xyq = -xz


Soln:
This equation can be written as the form

dx dy dz
 
P Q R

Manonmaniam Sundaranar University, Directorate of Distance & Continuing Education, Tirunelveli.


151
Where P=(y2 + z2) , Q = -xy and R = -xz.

The auxiliary equations are


dx dy dz
  .......... (1)
y  z  xy
2 2
xz
dy dz
Taking the last two equations, we get 
y z
Integrating we get log y = log z + constant.
y
  c1
z
Each of the equations (1) is equal to
xdx  ydy  zdz
x( y 2  z 2 )  xy 2  xz 2

xdx  ydy  zdz


i.e., .
0
 xdx  ydy  zdz  0.
Hence after integration this reduces x2 + y2 +z2 =c2.

Hence the general solution of the equation is


y 
F , x2  y2  z 2   0 .
z 

Ex.2 Find the general solution of (y+z)p + (z+x)q = x+y

Soln:

This equation can be written as the form

dx dy dz
 
P Q R
Where P= y + z , Q = z+x and R = x+y

Manonmaniam Sundaranar University, Directorate of Distance & Continuing Education, Tirunelveli.


152
dx dy dz
The auxiliary equations are  
yz zx x y

dx  dy  dz dx  dy dy  dz
Each is equal to  
2( x  y  z ) yx zy
Taking the first two, we get after integration
1
log(x+y+z) = -log (y-x) + constant
2
.˙. (x+y+z) (y-x)2 = C1
By taking the last two, we get
- log (y-x) = -log (z-y) + constant.
zy
 c2
yx
Hence the general solution of the equation is
 z  y
F (x + y + z) (y - x) 2 , 0
 y  x

z z
Ex. 3 Solve x 2  y2  ( x  y) z
x y

Soln: This equation can be written as the form

dx dy dz
 
P Q R
Where P= x2 , Q = y2 and R =( x+y)z.

The auxiliary equations are


dx dy dz
 2 
x 2
y ( x  y) z
dx  dy dz
i.e., 
x y
2 2
( x  y) z
dx  dy dz
i.e., 
x y z

Manonmaniam Sundaranar University, Directorate of Distance & Continuing Education, Tirunelveli.


153
i.e., log(x-y) = log z + constant.
x y
  c1
z
dx dy
Also 
x2 y2

Hence -(1/x) = -(1/y) + constant.


(1/y)-(1/x) = c2

1 1 x y
Hence the solution is F   ,   0 .
y x z 
Ex.4 Find the equation of the cone satisfying the equation xp + yq = z and passing through
the circle x2 + y2 + z2 =4,
Soln: Soln:
This equation can be written as the form

dx dy dz
 
P Q R
Where P= x , Q = y and R = z.

The auxiliary equations are


dx dy dz
 
x y z

Hence two independent solution of the equations are


x y
 a ----------- (i) and  b ------------ (ii)
y z

x y
.˙. The general solution of the equation is F  ,   0.
y z 

x y
Here we have to find a functional relation between and such that they also
y z
satisfy the equations
x2+y2+z2 = 4 ------- (iii) and x+y+z = 2 ---------- (iv)
Hence eliminate x,y,z from (i), (ii), (iii) and (iv)

Manonmaniam Sundaranar University, Directorate of Distance & Continuing Education, Tirunelveli.


154
x y x
y ,z  
a b ab
Substituting values of y,z in terms of x in (iii) and (iv), we get
x2 x2  1 1 
x2   4 i.e., x 2 1  2  2 2   4 .
a 2 a 2b 2  a ab 

x x  1 1 
x   2, i.e., x1     2 .
a ab  a ab 

2
1 x  1 1 
1  2  2 2  1   
a ab  a ab 

2 2 2
i.e.,   2 0
a ab a b

i.e., ab + a+ 1 = 0
Now if we replace a by (x/y) and b by (y/z), we get the required surface
xy + yz + zx = 0.

Ex. 5 Solve (x2 - yz)p + (y2-zx)q = z2 - xy.


Soln:

The subsidiary equations are

dx dy dz
 2  2
x  yz y  zx z  xy
2

dx  dy d ( x  y)

x  yz  ( y  zx) ( x  y)( x  y  z )
2 2

d ( y  z)

( y  z )( x  y  z )
d ( x  y) d ( y  z )
 
x y yz
Manonmaniam Sundaranar University, Directorate of Distance & Continuing Education, Tirunelveli.
155
x y
  c1 ............ (1)
yz
Using multipliers x,y,z each of the subsidiary equations
xdx  ydy  zdz

x  y 3  z 3  3xyz
3

𝑑𝑥 + 𝑑𝑦 + 𝑑𝑧
𝑎𝑛𝑑 𝑖𝑠 𝑎𝑙𝑠𝑜 𝑒𝑞𝑢𝑎𝑙 𝑡𝑜
𝑥2 + 𝑦2 + 𝑧2 − 𝑦𝑧 − 𝑧𝑥 − 𝑥𝑦

Taking the last two ratios 𝑥 𝑑𝑥 = ( 𝑥)𝑑 ( 𝑥).

( x  y  z)2 ( x 2  y 2  z 2 )
Integrating,   c2
2 2
.˙. xy + yz +zx = c2 …………..(2)
x y
From (1) and (2),  ( , xy  yz  zx)  0 ,where f is arbitrary.
yz
𝑥−𝑧
Note. We should not take the second solution as = 𝑐2 since each of the subsidiary
𝑧−𝑧

equations.
d ( z  x)
 
( z  x)( x  y  z )
𝑥−𝑦 x y
But this solution is not independent of the first solution = 𝑐 1 , since  1  c1  1
𝑧 −𝑥 yz
x y
gives  c1  1 which is merely the second solution.
yz

Excersices 28
z z
Solve given p  ; q :
x y
1. xp + yq = z.
2. xp - yq = xy
3. ap + bq + cz = 0
Manonmaniam Sundaranar University, Directorate of Distance & Continuing Education, Tirunelveli.
156
Prepared by
Dr. K. RUBIN MARY
Assistant Professor of Mathematics, St. Jude’s College,
Thoothoor - 629 176, Kanyakumari District.

Manonmaniam Sundaranar University, Directorate of Distance & Continuing Education, Tirunelveli.


157

You might also like